Buy BOOKS at Discounted Price

Differentiation

Class 12th Mathematics RD Sharma Volume 1 Solution
Exercise 11.1
  1. e-x Differentiate the following functions from first principles :…
  2. e3x Differentiate the following functions from first principles :…
  3. eax + b Differentiate the following functions from first principles :…
  4. ecos x Differentiate the following functions from first principles :…
  5. e^root 2x Differentiate the following functions from first principles :…
  6. log cos x Differentiate each of the following functions from the first…
  7. e^root cotx Differentiate each of the following functions from the first…
  8. x^2 ex Differentiate each of the following functions from the first principal :…
  9. log cosec x Differentiate each of the following functions from the first…
  10. sin-1(2x + 3) Differentiate each of the following functions from the first…
Very Short Answer
  1. If f(x) = loge (loge x), then write the value of f’(e).
  2. If f(x) = x + 1, then write the value of
  3. If f’ (1) = 2 and y = f(loge x), find . {dy}/{dx} . at x = e.…
  4. If f(1) = 4, f’(1) = 2, find the value of the derivative of log (f(ex)) with respect to…
  5. If f (x) = root { 2x^{2} - 1 } and y = f(x2), then find at x = 1.…
  6. Let g(x) be the inverse of an invertible function f(x) which is derivable at x = 3. If…
  7. If y = sin–1 (sin x), - { pi }/{2} less than equal to x leq frac { pi }/{2} Then…
  8. If { pi }/{2} less than equal to x leq frac { 3 pi }/{2} and y = sin–1 (sin x),…
  9. If π ≤ x ≤ 2π and y = cos–1 (cos x), find {dy}/{dx}
  10. If y = sin^{-1} ( {2x}/{ 1+x^{2} } ) write the value of {dy}/{dx} for x…
  11. If f(0) = f(1) = 0, f’(1) = 2 and y = f(ex) ef(x), write the value of {dy}/{dx}…
  12. If y = x|x|, find {dy}/{dx} for x 0.
  13. If y = sin–1 x + cos–1 x, find {dy}/{dx} .
  14. If x = a(θ + sin θ), y = a (1 + cos θ), find {dy}/{dx} .
  15. If - { pi }/{2} and y = tan^{-1}root { {1-cos2x}/{1+cos2x} } find…
  16. If y = xx, find {dy}/{dx} at x = e.
  17. If
  18. if y = loga x, find {dy}/{dx}
  19. If y = logroot {tanx} , {dy}/{dx}
  20. If y = sin^{-1} ( { 1-x^{2} }/{ 1+x^{2} } ) + cos^{-1} ( { 1-x^{2} }/{…
  21. If y = sec^{-1} ( {x+1}/{x-1} ) + sin^{-1} ( {x-1}/{x+1} ) then write the…
  22. If |x| 1 and y = 1 + x + x2 + … to ∞, then find the value of {dy}/{dx}…
  23. If u = sin^{-1} ( {2x}/{ 1+x^{2} } ) and v = tan^{-1} ( {2x}/{ 1+x^{2} } )…
  24. If f (x) = log { { u (x) }/{ v (x) } } , u (1) = v (1) and u’(1) = v’(1) = 2, then…
  25. If y = log |3x|, x ≠ 0, find {dy}/{dx}
  26. If f(x) is an even function, then write whether f’ (x) is even or odd.…
  27. If f(x) is an odd function, then write whether f’(x) is even or odd.…
  28. Write the derivative of sin x with respect to cos x.
Exercise 11.2
  1. sin(3x + 5) Differentiate the following functions with respect to x:…
  2. tan^2 x Differentiate the following functions with respect to x:
  3. tan(x° + 45°) Differentiate the following functions with respect to x:…
  4. sin(log x) Differentiate the following functions with respect to x:…
  5. e^sinroot x Differentiate the following functions with respect to x:…
  6. etan x Differentiate the following functions with respect to x:
  7. sin^2 (2x + 1) Differentiate the following functions with respect to x:…
  8. log7(2x - 3) Differentiate the following functions with respect to x:…
  9. tan(5x°) Differentiate the following functions with respect to x:…
  10. 2^x^3 Differentiate the following functions with respect to x:
  11. 3^e^x Differentiate the following functions with respect to x:
  12. logx3 Differentiate the following functions with respect to x:
  13. 3^x^2 + 2x Differentiate the following functions with respect to x:…
  14. root a^2 - x^2/a^2 + x^2 Differentiate the following functions with respect to…
  15. 3x log x Differentiate the following functions with respect to x:…
  16. root 1+sinx/1-sinx Differentiate the following functions with respect to x:…
  17. root 1-x^2/1+x^2 Differentiate the following functions with respect to x:…
  18. (log sin x)^2 Differentiate the following functions with respect to x:…
  19. root 1+x/1-x Differentiate the following functions with respect to x:…
  20. sin (1+x^2/1-x^2) Differentiate the following functions with respect to x:…
  21. e3x cos(2x) Differentiate the following functions with respect to x:…
  22. sin(log sin x) Differentiate the following functions with respect to x:…
  23. etan 3x Differentiate the following functions with respect to x:
  24. e^root cotx Differentiate the following functions with respect to x:…
  25. log (sinx/1+cosx) Differentiate the following functions with respect to x:…
  26. logroot 1-cosx/1+cosx Differentiate the following functions with respect to x:…
  27. tan(esin x) Differentiate the following functions with respect to x:…
  28. log (x + root x^2 + 1) Differentiate the following functions with respect to…
  29. e^xlogx/x^2 Differentiate the following functions with respect to x:…
  30. log(cosec x - cot x) Differentiate the following functions with respect to x:…
  31. e^ex + e^-2x/e^2x - e^-2x Differentiate the following functions with respect…
  32. log (x^2 + x+1/x^2 - x+1) Differentiate the following functions with respect…
  33. tan-1(ex) Differentiate the following functions with respect to x:…
  34. Differentiate the following functions with respect to x:
  35. sin(2sin-1x) Differentiate the following functions with respect to x:…
  36. e^tan^-1root x Differentiate the following functions with respect to x:…
  37. root tan^-1 (x/2) Differentiate the following functions with respect to x:…
  38. log(tan-1x) Differentiate the following functions with respect to x:…
  39. 2^xcosx/(x^2 + 3)^2 Differentiate the following functions with respect to x:…
  40. xsin(2x) + 5x + kk + (tan^2 x)^3 Differentiate the following functions with…
  41. log(3x + 2) - x^2 log(2x - 1) Differentiate the following functions with…
  42. 3x^2sinx/root 7-x^2 Differentiate the following functions with respect to x:…
  43. sin^2 {log(2x + 3)} Differentiate the following functions with respect to x:…
  44. ex log(sin 2x) Differentiate the following functions with respect to x:…
  45. root x^2 + 1 + root x^2 - 1/root x^2 + 1 - root x^2 - 1 Differentiate the…
  46. log x+2 + root x^2 + 4x+1 Differentiate the following functions with respect…
  47. (sin-1 x^4)^4 Differentiate the following functions with respect to x:…
  48. sin^-1 (x/root x^2 + a^2) Differentiate the following functions with respect…
  49. e^xsinx/(x^2 + 2)^3 Differentiate the following functions with respect to x:…
  50. 3e-3xlog(1 + x) Differentiate the following functions with respect to x:…
  51. x^2 + 2/root cosx Differentiate the following functions with respect to x:…
  52. x^2 (1-x^2)^3/cos2x Differentiate the following functions with respect to x:…
  53. log cot (pi /4 + x/2) Differentiate the following functions with respect to x:…
  54. eaxsec(x)tan(2x) Differentiate the following functions with respect to x:…
  55. log(cos x^2) Differentiate the following functions with respect to x:…
  56. cos(log x)^2 Differentiate the following functions with respect to x:…
  57. logroot x-1/x+1 Differentiate the following functions with respect to x:…
  58. If y = log root x-1 - root x+1 show that dy/dx = -1/2 root x^2 - 1 .…
  59. If y = root x+1 + root x-1 prove that root x^2 - 1 dy/dx = 1/2 y .…
  60. If y = x/x+2 prove that x dy/dx = (1-y) y .
  61. If y = log (root x + 1/root x) prove that dy/dx = x-1/2x (x+1) .
  62. If y = logroot 1+tanx/1-tanx prove that dy/dx = sec2x
  63. If y = root x + 1/root x prove that 2x dy/dx = root x - 1/root x
  64. If y = xsin^-1x/root 1-x^2 prove that (1-x^2) dy/dx = x + y/x .
  65. If y = e^x - e^-x/e^x + e^-x prove that dy/dx = 1-y^2 .
  66. If y = (x - 1)log (x - 1) - (x + 1) log (x +1), prove that dy/dx = log…
  67. If y = ex cos x, prove that dy/dx = root 2e^xcos (x + pi /4) .
  68. If y = 1/2 log (1-cos2x/1+cos2x) prove that dy/dx = 2cosec2x .
  69. If y = xsin^-1x + root 1-x^2 prove that dy/dx = sin^-1x .
  70. If y = root x^2 + a^2 prove that y dy/dx - x = 0 .
  71. If y = ex + e-x, prove that dy/dx = root y^2 - 4 .
  72. If y = root a^2 - x^2 prove that y dy/dx + x = 0 .
  73. If xy = 4, prove that x (dy/dx + y^2) = 3y .
  74. If prove that d/dx x/2 root a^2 - x^2 + a^2/2 sin^-1 x/a = root a^2 - x^2 .…
Exercise 11.3
  1. cos^-1 2x root 1-x^2 , 1/root 2 x1 Differentiate the following functions with…
  2. cos^-1 root 1+x/2 ,-1x1 Differentiate the following functions with respect to…
  3. sin^-1 root 1-x/2 , 0x1 Differentiate the following functions with respect to…
  4. sin^-1 root 1-x^2 , 0x1 Differentiate the following functions with respect to…
  5. tan^-1 x/root a^2 - x^2 , axa Differentiate the following functions with…
  6. sin^-1 x/root x^2 + a^2 Differentiate the following functions with respect to…
  7. sin-1 (2x^2 - 1), 0 x 1 Differentiate the following functions with respect to…
  8. sin-1 (1 - 2x^2), 0 x 1 Differentiate the following functions with respect to…
  9. cos^-1 x/root x^2 + a^2 Differentiate the following functions with respect to…
  10. sin^-1 sinx+cosx/root 2 , - 3 pi /4 x pi /4 Differentiate the following…
  11. cos^-1 cosx+sinx/root 2 , pi /4 x pi /4 Differentiate the following functions…
  12. tan^-1 x/1 + root 1-x^2 ,-1x1 Differentiate the following functions with…
  13. tan^-1 x/a + root a^2 - x^2 ,-axa Differentiate the following functions with…
  14. sin^-1 x + root 1-x^2/root 2 ,-1x1 Differentiate the following functions with…
  15. cos^-1 x + root 1-x^2/root 2 ,-1x1 Differentiate the following functions with…
  16. tan^-1 4x/1-4x^2 , - 1/2 x 1/2 Differentiate the following functions with…
  17. tan^-1 (2^x+1/1-4^x) , - infinity x0 Differentiate the following functions…
  18. tan^-1 (2a^x/1-a^2x) , a1 , - infinity x0 Differentiate the following…
  19. sin^-1 root 1+x + root 1-x/2 , 0x1 Differentiate the following functions with…
  20. tan^-1 root 1+a^2x^2 - 1/ax , x not equal 0 Differentiate the following…
  21. tan^-1 (sinx/1+cosx) , - pi x pi Differentiate the following functions with…
  22. sin^-1 (1/root 1+x^2) Differentiate the following functions with respect to x:…
  23. cos^-1 (1-x^2n/1+x^2n) , 0x infinity Differentiate the following functions…
  24. sin^-1 (1-x^2/1+x^2) + sec^-1 (1+x^2/1-x^2) , x inr Differentiate the…
  25. tan^-1 (a+x/1-ax) Differentiate the following functions with respect to x:…
  26. tan^-1 (root x + root a/1 - root xa) Differentiate the following functions…
  27. tan^-1 (a+btanx/b-atanx) Differentiate the following functions with respect to…
  28. tan^-1 (a+bx/b-ax) Differentiate the following functions with respect to x:…
  29. tan^-1 (x-a/x+a) Differentiate the following functions with respect to x:…
  30. tan^-1 (x/1+6x^2) Differentiate the following functions with respect to x:…
  31. tan^-1 5x/1-6x^2 , - 1/root 6 x 1/root 6 Differentiate the following functions…
  32. tan^-1 cosx+sinx/cosx-sinx , - pi /4 x pi /4 Differentiate the following…
  33. tan^-1 x^1/3+a^1/3/1 - (ax)^1/3 Differentiate the following functions with…
  34. sin^-1 (2^x+1/1+4^x) Differentiate the following functions with respect to x:…
  35. If y = sin^-1 (2x/1+x^2) + sec^-1 (1+x^2/1-x^2) 0 x 1, prove that dy/dx =…
  36. If y = sin^-1 (x/root 1+x^2) + cos^-1 (1/root 1+x^2) 0 x ∞, prove that dy/dx =…
  37. cos-1 (sin x) Differentiate the following with respect to x:
  38. cot^-1 (1-x/1+x) Differentiate the following with respect to x:
  39. If y = cot^-1 root 1+sinx + root 1-sinx/root 1+sinx - root 1-sinx show that…
  40. If y = tan^-1 (2x/1-x^2) + sec^-1 (1+x^2/1-x^2) x 0, prove that dy/dx =…
  41. If y = sec^-1 (x+1/x-1) + sin^-1 (x-1/x+1) x 0. Find dy/dx .
  42. If y = sin[2tan^-1 root 1-x/1+x] find dy/dx .
  43. If y = cos^-1 (2x) + 2cos^-1root 1-4x^2 0x 1/2 find dy/dx .
  44. If the derivative of tan-1 (a + bx) takes the value 1 at x = 0, prove that 1 +…
  45. If y = cos-1 (2x) + 2 cos-1 root 1-4x^2 x 0, find dy/dx .
  46. If y = tan^-1 root 1+x - root 1-x/root 1+x + root 1-x find dy/dx .…
  47. If y = cos^-1 2x-3 root 1-x^2/root 13 find dy/dx .
  48. Differentiate sin^-1 2^x+1 3^x/1 + (36)^x with respect to x.
  49. If y = sin^-1 (6x root 1-9x^2) , - 1/3 root 2 x 1/3 root 2 then find dy/dx .…
Exercise 11.4
  1. Find dy/dx in each of the following: xy = c^2
  2. Find dy/dx in each of the following: y^3 - 3xy^2 = x^3 + 3x^2 y
  3. Find dy/dx in each of the following: x2/3 + y2/3 = a2/3
  4. Find dy/dx in each of the following: 4x + 3y = log (4x - 3y)
  5. Find dy/dx in each of the following: x^2/a^2 + y^2/b^2 = 1
  6. Find dy/dx in each of the following: x^5 + y^5 = 5xy
  7. Find dy/dx in each of the following: (x + y)^2 = 2axy
  8. Find dy/dx in each of the following: (x^2 + y^2)^2 = xy
  9. Find dy/dx in each of the following: tan - 1 (x^2 + y^2) = a
  10. Find dy/dx in each of the following: e^x-y = log (x/y)
  11. Find dy/dx in each of the following: sinxy + cos (x + y) = 1
  12. If root 1-x^2 + root 1-y^2 = a (x-y) prove that dy/dx = root 1-y^2/1-x^2…
  13. If y root 1-x^2 + x root 1-y^2 = 1 prove that dy/dx = root 1-y^2/1-x^2…
  14. If xy = 1, prove that dy/dx + y^2 = 0
  15. If xy^2 = 1, prove that 2 dy/dx + y^3 = 0
  16. If root 1+y+y root 1+x = 0 prove that (1+x)^2 dy/dx + 1 = 0
  17. If logroot x^2 + y^2 = tan^-1 (y/x) prove that dy/dx = x+y/x-y
  18. If sec (x+y/x-y) = a prove that dy/dx = y/x
  19. If tan^-1 (x^2 - y^2/x^2 + y^2) = a prove that dy/dx = x/y (1-tana)/(1+tana)…
  20. If xy log (x + y) = 1, prove that dy/dx = y (x^2y+x+y)/x (xy^2 + x+y)…
  21. If y = x sin (a + y), prove that dy/dx = sin^2 (a+y)/sin (a+y) - ycos (a+y)…
  22. If x sin (a + y) + sin a cos (a + y) = 0, prove that dy/dx = sin^2 (a+y)/sina…
  23. If y - x sin y, prove that dy/dx = siny/(1-xcosy)
  24. If y root x^2 + 1 = log (root x^2 + 1 - x) show that (x^2 + 1) dy/dx + xy+1 =…
  25. If sin (xy) + y/x = x^2 - y^2 find dy/dx .
  26. If tan (x + y) + tan (x - y) = 1, find dy/dx .
  27. If ex + ey = ex + y, prove that dy/dx = - e^x (e^y - 1)/e^y (e^x - 1) or,…
  28. If cos y = x cos(a + y), with cos a ≠±1, prove that dy/dx = cos^2 (a+y)/sina…
  29. If sin^2 y + cosxy = k, find dy/dx at x = 1, y = pi /4
  30. If y = log_cosx sinx log_sinx cosx^-1 + sin^-1 (2x/1+x^2) find dy/dx at x = pi…
  31. If root y+x + root y-x = c show that dy/dx = y/x - root y^2/x^2 - 1…
Exercise 11.5
  1. x1/x Differentiate the following functions with respect to x :
  2. xsin x Differentiate the following functions with respect to x :
  3. (1 + cos x)x Differentiate the following functions with respect to x :…
  4. x^cos^-1x Differentiate the following functions with respect to x :…
  5. (log x)x Differentiate the following functions with respect to x :…
  6. (log x)cos x Differentiate the following functions with respect to x :…
  7. (sin x)cos x Differentiate the following functions with respect to x :…
  8. ex log x Differentiate the following functions with respect to x :…
  9. (sin x)log x Differentiate the following functions with respect to x :…
  10. 10log sin x Differentiate the following functions with respect to x :…
  11. (log x)log x Differentiate the following functions with respect to x :…
  12. 10(10x) Differentiate the following functions with respect to x :…
  13. sin (xx) Differentiate the following functions with respect to x :…
  14. (sin-1 x)x Differentiate the following functions with respect to x :…
  15. x^sin^-1x Differentiate the following functions with respect to x :…
  16. (tan x)1/x Differentiate the following functions with respect to x :…
  17. x^tan^-1x Differentiate the following functions with respect to x :…
  18. (x^x) root x Differentiate the following functions with respect to x :…
  19. x^(sinx-cosx) + x^2 - 1/x^2 + 1 Differentiate the following functions with…
  20. x^xcosx + x^2 + 1/x^2 - 1 Differentiate the following functions with respect…
  21. (x cos x)x + (x sin x)1/x Differentiate the following functions with respect…
  22. (x + 1/x)^x + x^(1 + 1/x) Differentiate the following functions with respect…
  23. esin x + (tan x)x Differentiate the following functions with respect to x :…
  24. (cos x)x + (sin x)1/x Differentiate the following functions with respect to x…
  25. x^x^2 - 3 + (x-3)^x^2 Differentiate the following functions with respect to x…
  26. Find dy/dx , when 1y = ex + 10x + xx
  27. Find dy/dx , when y = xn + nx + xx + nn
  28. Find dy/dx , when y = (x^2 - 1)^3 (2x-1)/root (x-3) (4x-1)
  29. Find dy/dx , when y = e^2xsexxlogx/root 1-2x
  30. Find dy/dx , when y = e3x sin 4x 2x
  31. Find dy/dx , when y = sin x sin 2x sin 3x sin 4x
  32. Find dy/dx , when y = xsin x + (sin x)x
  33. Find dy/dx , when y = (sin x)cos x + (cos x)sin x
  34. Find dy/dx , when y = (tan x)cot x + (cot x)tan x
  35. Find dy/dx , when y = (sinx)^x + sin^-1root x
  36. Find dy/dx , when y = xcos x + (sin x)tan x
  37. Find dy/dx , when y = xx + (sin x)x
  38. Find dy/dx , when y = (tan x)log x + cos^2 (pi /4)
  39. Find , when y = xx + x1/x
  40. Find dy/dx , when y = xlog x + (log x)x
  41. If x^13 y^7 = (x + y)^20 , prove that dy/dx = y/x
  42. If x^16 y^9 = (x^2 + y)^17 , prove that
  43. If y = sin (xx), prove that dy/dx = cos (x^x) x^x (1+logx)
  44. If xx + yx = 1, prove that dy/dx = - x^x (1+logx) + y^x logy/x y^(x-1)…
  45. If xx + yx = 1, find dy/dx = - y (y+xlogy)/x (ylogx+x)
  46. If xy + yx = (x + y)x + y, find dy/dx
  47. If xmyn = 1, prove that dy/dx = - my/nx
  48. If yx = ey - x prove that dy/dx = (1+logy)^2/logy
  49. If (sin x)y = (cos y)x, prove that dy/dx = logcosy-ycotx/logsinx+xtany…
  50. If (cos x)y = (tan y)x, prove that dy/dx = logtany+ytanx/logcosx-xsecycosecy…
  51. If ex + ey = ex + y, prove that
  52. If ey = yx, prove that
  53. If ex + y - x = 0, prove that dy/dx = 1-x/x
  54. If y = x sin(a + y), prove that
  55. If x sin (a + y) + sin a cos (a + y) = 0, prove that dy/dx = sin^2 (a+y)/sina…
  56. If (sin x)y = x + y, prove that dy/dx = 1 - (x+y) ycotx/(x+y) logsinx-1…
  57. If xy log(x + y) = 1, prove that dy/dx = y (x^2y+x+y)/x (xy^2 + x+y)…
  58. If y = x sin y, prove that
  59. Find the derivative of the function f(x) given by f(x) = (1 + x)(1 + x^2)(1 +…
  60. If y = log x^2 + x+1/x^2 - x+1 + 2/root 3 tan^-1 (root 3x/1-x^2) find dy/dx .…
  61. If y = (sinx-cosx)^sinx-cosx , pi /4 x 3 pi /4 find dy/dx .
  62. If xy = ex - y, find dy/dx .
  63. If yx + xy + xx = ab, find dy/dx .
  64. If (cos x)y = (cos y)x find dy/dx .
  65. If cos y = x cos (a + y), where cosanot equal plus or minus 1 prove that dy/dx…
  66. If (x-y) e^x/x-y = a prove that:
  67. If x = e^x/y prove that dy/dx = x-y/xlogx
  68. If y = x^tanx + root x^2 + 1/2 find dy/dx
  69. If y = 1 + alpha /(1/x - alpha) + beta /x/(1/x - alpha) (1/x - beta) + gamma…
Exercise 11.6
  1. If y = root x + root x + root x + l t0 prove that dy/dx = 1/2y-1
  2. If y = root cosx + root cosx + root cosx + l t_0 prove that dy/dx = sinx/1-2y…
  3. If y = root logx + root logx + root logx + l .t0 infinity prove that (2y-1)…
  4. If y = root tanx + root tanx + root tanx + l t0 prove that dy/dx = sec^2x/2y-1…
  5. If y = (sinx)^(sinx) (sinx)^- infinity prove that dy/dx = y^2cotx/(1-ylogsinx)…
  6. If y = (tanx)^(tanx)^(tanx) prove that dy/dx = 2 at x = pi /4
  7. If y = e^x^e^x + x^e^x^x + e^x^e^e prove that dy/dx = e^x^x x^e^x e^x/x+e^x…
  8. If y (cosx)^(cosx) (cosx) l infinity prove that dy/dx = y^2tanx/(1-ylogcosx)…
Exercise 11.7
  1. Find dy/dx , when x = at^2 and y = 2at
  2. Find dy/dx , when x = a(θ + sinθ) and y = a(1 - cosθ)
  3. Find dy/dx , when x = acosθ and y = bsinθ
  4. Find dy/dx , when x = aeθ (sin θ - cos θ),y = aeθ(sinθ + cosθ)
  5. Find dy/dx , when x = b sin^2 θ and y = a cos^2 θ
  6. Find dy/dx , when x = a(1 - cos θ) and y = a(θ + sin θ) at theta = pi /2…
  7. Find dy/dx , when x = e^t + e^-t/2 and y = e^t - e^-t/2
  8. Find dy/dx , when x = 3/1+t^2 and y = 3at^2/1+t^2
  9. Find dy/dx , when x = a(cosθ + θ sinθ) and y = a(sinθ - cosθ)
  10. Find dy/dx , when x = e^6 (theta + 1/theta) and y = e^- theta (theta -…
  11. Find dy/dx , when x = 2t/1+t^2 and y = 1-t^2/1+t^2
  12. Find dy/dx , when x = cos^-1 1/root 1+t^2 and y = sin^-1 t/root 1+t^2 , t inr…
  13. Find dy/dx , when x = 1-t^2/1+t^2 and y = 2t/1+t^2
  14. Find dy/dx , when If x = 2cos θ - cos 2θ and y = 2sin θ - sin 2θ, prove that…
  15. Find dy/dx , when If x = ecos 2 t and y = esin 2t, prove that dy/dx = -…
  16. Find dy/dx , when If x = cos t and y = sin t, prove that dy/dx = 1/root 3 at 1…
  17. Find dy/dx , when If x = a (t + 1/t) and y = a (t - 1/t) prove that dy/dx =…
  18. Find dy/dx , when If x = sin^-1 (2t/1+t^2) and y = tan^-1 (2t/1-t^2) -1 t 1,…
  19. Find dy/dx , when If x = sin^3t/root cos2t , y = cos^3t/root cos2t find dy/dx…
  20. Find dy/dx , when If x = (t + 1/t)^2 , y = a^t + 1/i find dy/dx
  21. Find dy/dx , when If x = a (1+t^2/1-t^2) and y = 2t/1-t^2 find dy/dx…
  22. Find dy/dx , when If x = 10 (t - sin t), y = 12 (1 - cos t), find dy/dx .…
  23. Find dy/dx , when If x = a(θ - sin θ) and y = a (1 + cos θ), find dy/dx at…
  24. Find dy/dx , when If x = a sin 2t (1 + cos 2t) and y = b cos 2t (1 - cos 2t),…
  25. Find dy/dx , when If x = cos t (3 - 2 cos^2 t) and y = sin t (3 - 2 sin^2 t)…
  26. Find dy/dx , when If x = 1+logt/t^2 , y = 3+2logt/t find dy/dx
  27. Find dy/dx , when If x = 3 sin t - sin 3t, y = 3 cost - cos 3t, find dy/dx at…
  28. Find dy/dx , when If sinx = 2t/1+t^2 , tany = 2t/1-t^2 find dy/dx…
Exercise 11.8
  1. Differentiate x^2 with respect to x^3 .
  2. Differentiate log(1 + x^2) with respect to tan-1x.
  3. Differentiate (log x)x with respect to log x.
  4. Differentiate sin^-1root 1-x^2 with respect to cos-1 x, if x ϵ (0, 1)…
  5. Differentiate sin^-1root 1-x^2 with respect to cos-1 x, if x ϵ (-1, 0)…
  6. Differentiate sin^-1 (4x root 1-4x^2) with respect to root 1-4x^2 if x in (-…
  7. Differentiate sin^-1 (4x root 1-4x^2) with respect to root 1-4x^2 if x in (1/2…
  8. Differentiate sin^-1 (4x root 1-4x^2) with respect to root 1-4x^2 if x in (-…
  9. Differentiate tan^-1 (root 1+x^2 - 1/x) with respect to sin^-1 (2x/1+x^2) if…
  10. Differentiate sin^-1 (2x root 1-x^2) with respect to sec^-1 (1/root 1-x^2) if…
  11. Differentiate sin^-1 (2x root 1-x^2) with respect to sec^-1 (1/root 1-x^2) if…
  12. Differentiate (cos x)sin x with respect to (sin x)cos x.
  13. Differentiate sin^-1 (2x/1+x^2) with respect to cos^-1 (1-x^2/1+x^2) if 0 x 1.…
  14. Differentiate tan^-1 (1+ax/1-ax) with respect to root 1+a^2x^2
  15. Differentiate sin^-1 (2x root 1-x^2) with respect to tan^-1 (x/root 1-x^2) if…
  16. Differentiate tan^-1 (2x/1-x^2) with respect to cos^-1 (1-x^2/1+x^2) if 0 x 1.…
  17. Differentiate tan^-1 (x-1/x+1) with respect to sin^-1 (3x-4x^3) if - 1/2 x 1/2…
  18. Differentiate tan^-1 (cosx/1+sinx) with respect to sec-1 x.
  19. Differentiate sin^-1 (2x/1+x^2) with respect to tan^-1 (2x/1-x^2) if -1 x 1.…
  20. Differentiate cos^-1 (4x^3 - 3x) with respect to tan^-1 (1-x^2/x) if 1/2 x1 .…
  21. Differentiate tan^-1 (x/root 1-x^2) with respect to sin^-1 (2x root 1-x^2) if…
  22. Differentiate sin^-1root 1-x^2 with respect to cot^-1 (x/root 1-x^2) if 0 x 1.…
  23. Differentiate sin^-1 (2axroot 1-a^2x^2) with respect to root 1-a^2x^2 if -…
  24. Differentiate tan^-1 (1-x/1+x) with respect to root 1-x^2 if -1 x 1.…
Mcq
  1. If f(x) = logx2 (log x), then f’(x) at x = e is Choose the correct alternative in the…
  2. The differential coefficient of f(log x) with respect to x, where f(x) = log x is Choose…
  3. The derivative of the function cos^{-1} { (cos2x)^{1/2} } at x = π/6 is Choose the…
  4. Differential coefficient of sec (tan–1 x) is Choose the correct alternative in the…
  5. If f(x) = tan–1 root { {1+sinx}/{1-sinx} } 0 ≤ x ≤ π/2, then f’ (π/6) is Choose…
  6. If y = ( 1 + {1}/{x} ) ^{x} , frac {dy}/{dx} = Choose the correct alternative in the…
  7. If xy = ex–y, then {dy}/{dx} is Choose the correct alternative in the following:…
  8. Given f(x) = 4x8, then Choose the correct alternative in the following:…
  9. If x = a cos3 θ, y = a sin3 θ, then root { 1 + ( {dy}/{dx} ) ^{2} } = Choose the…
  10. If Choose the correct alternative in the following:
  11. The derivative of sec^{-1} ( {1}/{ 2x^{2} + 1 } ) with respect to root {1+3x} at x…
  12. For the curve . root {x} + sqrt{y} = 1 , {dy}/{dx} . at (1/4, 1/4) is Choose the…
  13. If sin (x + y) = log (x + y), then {dy}/{dx} = Choose the correct alternative in…
  14. Let u = sin^{-1} ( {2x}/{ 1+x^{2} } ) and v = tan^{-1} ( {2x}/{ 1+x^{2} } ) , frac…
  15. {d}/{dx} { tan^{-1} ( frac {cosx}/{1+sinx} ) } equals Choose the correct alternative…
  16. {d}/{dx} [ log { e^{x} ( frac {x-2}/{x+2} ) ^{3/4} } ] equals Choose the correct…
  17. If y = root {sinx+y} , {dy}/{dx} = Choose the correct alternative in the following:…
  18. If 3 sin(xy) + 4cos (xy) = 5, then {dy}/{dx} = Choose the correct alternative in…
  19. If sin y = x sin (a + y), then {dy}/{dx} is Choose the correct alternative in…
  20. The derivative of cos–1 (2x2 – 1) with respect to cos–1 x is Choose the correct…
  21. If f (x) = root { x^{2} + 6x+9 } then f’(x) is equal to Choose the correct…
  22. If f(x) = |x2 – 9x + 20|, then f’(x) is equal to Choose the correct alternative in the…
  23. If f (x) = root { x^{2} - 10x+25 } then the derivative of f(x) in the interval [0,…
  24. If f(x) = |x – 3| and g(x) = fof(x), then for x 10, g’(x) is equal to Choose the…
  25. If then f’(x) is equal to Choose the correct alternative in the following:…
  26. If, y = {1}/{ 1+x^{2-b} + x^{c-b} } + frac {1}/{ 1+x^{b-c} + x^{2-c} } . +…
  27. If root { 1-x^{6} } + sqrt { 1-y^{6} } = a^{3} ( x^{3} - y^{3} ) then…
  28. If y = logroot {tanx} then the value of {dy}/{dx} at x = { pi }/{4} is…
  29. If sin^{-1} ( { x^{2} - y^{2} }/{ x^{2} + y^{2} } ) = loga frac {dy}/{dx} is…
  30. If sin y = x cos(a + y), then {dy}/{dx} is equal to Choose the correct…
  31. If y = log ( { 1-x^{2} }/{ 1+x^{2} } ) , frac {dy}/{dx} = Choose the correct…
  32. If y = root {sinx+y} , {dy}/{dx} equals. Choose the correct alternative in the…
  33. If y = tan^{-1} ( {sinx+cosx}/{cosx-sinx} ) then {dy}/{dx} is equal to…

Exercise 11.1
Question 1.

Differentiate the following functions from first principles :

e–x


Answer:

We have to find the derivative of e–x with the first principle method, so,

f(x) = e–x


by using the first principle formula, we get,


f ‘(x) =


f ‘(x) =


f ‘(x) =


f ‘(x) =


[By using = 1]


f ‘(x) = – e–x



Question 2.

Differentiate the following functions from first principles :

e3x


Answer:

We have to find the derivative of e3x with the first principle method, so,

f(x) = e3x


by using the first principle formula, we get,


f ‘(x) =


f ‘(x) =


f ‘(x) =


f ‘(x) =


[By using = 1]


f ‘(x) = 3e3x



Question 3.

Differentiate the following functions from first principles :

eax + b


Answer:

We have to find the derivative of eax+b with the first principle method, so,

f(x) = eax+b


by using the first principle formula, we get,


f ‘(x) =


f ‘(x) =


f ‘(x) =


[By using = 1]


f ‘(x) = a eax+b



Question 4.

Differentiate the following functions from first principles :

ecosx


Answer:

We have to find the derivative of ecosx with the first principle method, so,

f(x) = ecosx


by using the first principle formula, we get,


f ‘(x) =


f ‘(x) =


f ‘(x) =


f ‘(x) =


[By using = 1]


f ‘(x) =


f ‘(x) =


[By using cos(x+h) = cosx cosh – sinx sinh]


f ‘(x) =


[By using limx0 = 1 and


cos 2x = 1–2sin2 x]


f ‘(x) =


f ‘(x) =


f ‘(x) = –ecos x sin x



Question 5.

Differentiate the following functions from first principles :



Answer:

We have to find the derivative of e√2x with the first principle method, so,

f(x) = e2x


by using the first principle formula, we get,


f ‘(x) =


f ‘(x) =


f ‘(x) =


f ‘(x) =


[By using = 1]


f ‘(x) =


[By rationalising]


f ‘(x) =


f ‘(x) =



Question 6.

Differentiate each of the following functions from the first principal :

log cos x


Answer:

We have to find the derivative of log cosx with the first principle method, so,

f(x) = log cos x


by using the first principle formula, we get,


f ‘(x) =


f ‘(x) =


f ‘(x) =


f ‘(x) =


[Adding and subtracting 1]


f ‘(x) =


[Rationalising]


f ‘(x) =


[By using limx0 = 1]


f ‘(x) =


[cosC – cosD = –2 sin sin]


f ‘(x) = [By using limx0 = 1]


f ‘(x) =


f ‘(x) = – tan x



Question 7.

Differentiate each of the following functions from the first principal :



Answer:

We have to find the derivative of with the first principle method, so,

f(x) =


by using the first principle formula, we get,


f ‘(x) =


f ‘(x) =


f ‘(x) =


f ‘(x) =


[By using limx0 = 1]


f ‘(x) =


[Rationalizing]


f ‘(x) =


f ‘(x) =


[sinA cosB – cosA sinB = sin(A–B)]


f ‘(x) =


[By using limx0 = 1]


f ‘(x) =


f ‘(x) =



Question 8.

Differentiate each of the following functions from the first principal :

x2 ex


Answer:

We have to find the derivative of x2ex with the first principle method, so,

f(x) = x2ex


by using the first principle formula, we get,


f ‘(x) =


f ‘(x) =


f ‘(x) =


[By using (a+b)2 = a2+b2+2ab]


f ‘(x) =


f ‘(x) =


[By using limx0 = 1]


f ‘(x) = x2ex + limh0 e(x+h) [h+2x]


f ‘(x) = x2ex + 2x ex



Question 9.

Differentiate each of the following functions from the first principal :

log cosec x


Answer:

We have to find the derivative of log cosec x with the first principle method, so,

f(x) = log cosecx


by using the first principle formula, we get,


f ‘(x) =


f ‘(x) =


f ‘(x) =


f ‘(x) =


[By using log a – log b = log ]


f ‘(x) =


[adding and subtracting 1]


f ‘(x) =


f ‘(x) =


[Rationalising]


f ‘(x) =


f ‘(x) =


[sin C – sin D = 2 sin cos ]


f ‘(x) =


[By using limx0 = 1]


f ‘(x) = – cot x



Question 10.

Differentiate each of the following functions from the first principal :

sin–1(2x + 3)


Answer:

We have to find the derivative of sin–1(2x+3) with the first principle method, so,

f(x) = sin–1(2x+3)


by using the first principle formula, we get,


f ‘(x) =


f ‘(x) =


Let sin–1[2(x+h)+3] = A and sin–1(2x+3) = B, so,


sinA = [2(x+h)+3] and sinB = (2x+3),


2h = sinA – sinB, when h→0 then sinA→sinB we can also say that AB and hence A–B→0,


f ‘(x) =


f ‘(x) =


[sinC – sinD = 2 sin cos ]


f ‘(x) =


[By using limx0 = 1]


f ‘(x) =


f ‘(x) =


[By using Pythagoras theorem, in which H = 1 and P = 2x+3, so, we have to find B, which comes out to be by the relation H2 = P2 + B2]


f ‘(x) =




Very Short Answer
Question 1.

If f(x) = loge (loge x), then write the value of f’(e).


Answer:

f(x) = loge(logex)


Using the Chain Rule of Differentiation,



So,


(Ans)



Question 2.

If f(x) = x + 1, then write the value of


Answer:

f(x) = x + 1


(fof)(x) = f(x) + 1


= (x + 1) + 1


= x + 2


So,


=1 (Ans)



Question 3.

If f’ (1) = 2 and y = f(loge x), find .. at x = e.


Answer:

y = f(logex)


Using the Chain Rule of Differentiation,



So, at x = e




(Ans)



Question 4.

If f(1) = 4, f’(1) = 2, find the value of the derivative of log (f(ex)) with respect to x at the point x = 0.


Answer:

Using the Chain Rule of Differentiation, derivative of log(f(ex)) w.r.t. x is


So, the value of the derivative at x = 0 is





So, the value of the derivative at x = 0 is 0.5 (Ans)



Question 5.

If and y = f(x2), then find at x = 1.


Answer:

y = f(x2)





Putting x = 1,




=2√1


=2


i.e., at x = 1. (Ans)



Question 6.

Let g(x) be the inverse of an invertible function f(x) which is derivable at x = 3. If f(3) = 9 and f’(3) = 9, write the value of g’(9).


Answer:

From the definition of invertible function,


g(f(x)) = x …(i)


So, g(f(3)) = 3, i.e., g(9) = 3


Now, differentiating both sides of equation (i) w.r.t. x using the Chain Rule of Differentiation, we get –


g’(f(x)). f’(x) = 1 …(ii)


Plugging in x = 3 in equation (ii) gives us –


g’(f(3)).f’(3) = 1


or, g’(9).9 = 1


i.e., g’(9) = 1/9 (Ans)



Question 7.

If y = sin–1 (sin x), Then write the value of .


Answer:

For x,



=x


So, (Ans)



Question 8.

If and y = sin–1 (sin x), find


Answer:

For ,


y=sin-1 (sin x)


= sin-1 ( sin (π –(π -x))


(to get y in principal range of sin-1 x)


i.e.,


y = π - x



From the last problem we see that and


So, y is not differentiable at


Extending this, we can say that y is not differentiable at x = (2n+1)


So, for


(Ans)



Question 9.

If π ≤ x ≤ 2π and y = cos–1 (cos x), find


Answer:

y = cos-1 (cos x)


for x (π, 2π)


y = cos-1(cos x)


= cos-1(cos (π + (x - π)))


= cos-1(-cos (x-π))


= π – (x - π)


= 2π - x


[Since, cos(π+x) = - cos x and cos-1(-x) = π-x]


So,


For cos-1(cos x), x = n are the ‘sharp corners’ where slope changes from 1 to -1 or vice versa, i.e., the points where the curves are not differentiable.


So, for x [π,2π]


(Ans)



Question 10.

If write the value of for x > 1.


Answer:


So,




So, answer is (Ans)



Question 11.

If f(0) = f(1) = 0, f’(1) = 2 and y = f(ex) ef(x), write the value of at x = 0.


Answer:


Using the Chain Rule of Differentiation,



= f(ex). ef(x) f(x) + f(ex)ex. ef(x)


At x = 0,



= f(1). ef(0) f’(0) + f’(1). ef(0)


= 0. e0 f’(0) + 2.e0


= 0 + 2.1


= 2



Question 12.

If y = x|x|, find for x < 0.


Answer:

y = x|x|


or,


So, for x < 0



=-2x (Ans)



Question 13.

If y = sin–1 x + cos–1 x, find .


Answer:

We know that


So, here y = sin–1 x + cos–1 x


which is a constant.


Also, sin-1 x and cos-1 x exist only when -1 x 1


So, when x [-1, 1] and does not exist for all other values of x.



Question 14.

If x = a(θ + sin θ), y = a (1 + cos θ), find .


Answer:

and


Using Chain Rule of Differentiation,









=cot θ-cosec θ (Ans)



Question 15.

If and find .


Answer:





When , tan x is negative. So, square root of tan2 x in this condition is –tan x.


So,


=tan-1 (-tan x)


=-tan-1 (tan x)


=-x


And so


(Ans)



Question 16.

If y = xx, find at x = e.


Answer:

y = xx


Taking logarithm on both sides,


log y = x log x


Differentiating w.r.t. x on both sides,



=1+log x



=xx (1+log x)


So, at x = e,



=ee (1+1)


=2ee (Ans)



Question 17.

If


Answer:


Using the Chain Rule of Differentiation,





(Ans)



Question 18.

if y = loga x, find


Answer:

y = loga x =



(Ans)



Question 19.

If


Answer:

This particular problem is a perfect way to demonstrate how simple but powerful the Chain Rule of Differentiation is.


It is important to identify and break the problem into the individual functions with respect to which successive differentiation shall be done.


In this case, this is the way to break down the problem –



i.e.,





(Ans)



Question 20.

If find


Answer:

holds for all .


So, , for all


( )


Hence, for all .



Question 21.

If then write the value of


Answer:



Which exists for and is equal to


Now,






⟹x+1>0


⟹x>-1 …(i)


Also,





⟹x≥0 or x<-1 …(ii)


Comparing equations (i) and (ii), we understand that the condition satisfying both inequalities is .


So, for x≥0,


, which is a constant


So, (Ans)



Question 22.

If |x| < 1 and y = 1 + x + x2 + … to ∞, then find the value of


Answer:

Since |x| < 1,


y = 1 + x + x2 + … to ∞




(Ans)



Question 23.

If and where –1 < x < 1, then write the value of


Answer:

and


We know,


Using the chain rule of differentiation,





Using Chain Rule of Differentiation,





Dividing numerator and denominator by (1+x2)2,




=sec u (1+tan u ) (Ans)



Question 24.

If (1) and u’(1) = v’(1) = 2, then find the value of f’(1).


Answer:

Using the Chain Rule of Differentiation,




Putting x = 1,




Since, u(1) = v(1),


2v(1) – 2u(1) = 0


i.e., f’(1) = 0 (Ans)



Question 25.

If y = log |3x|, x ≠ 0, find


Answer:

y = log |3x|


So,



i.e.,; (Ans)



Question 26.

If f(x) is an even function, then write whether f’ (x) is even or odd.


Answer:

f(x) is an even function.


This means that f(-x) = f(x).


If we differentiate this equation on both sides w.r.t. x, we get –


f’(-x).(-1) = f’(x)


or, -f’(-x) = f’(x)


i.e., f’(x) is an odd function. (Ans)



Question 27.

If f(x) is an odd function, then write whether f’(x) is even or odd.


Answer:

f(x) is an odd function.


This means that f(-x) = -f(x).


If we differentiate this equation on both sides w.r.t. x, we get –


f’(-x).(-1) = -f’(x)


or, f’(-x) = f’(x)


i.e., f’(x) is an even function. (Ans)



Question 28.

Write the derivative of sin x with respect to cos x.


Answer:

We have to find


So, we use the Chain Rule of Differentiation to evaluate this.




=-cot x (Ans)




Exercise 11.2
Question 1.

Differentiate the following functions with respect to x:

sin(3x + 5)


Answer:

Let y = sin(3x + 5)


On differentiating y with respect to x, we get



We know


[using chain rule]




However, and derivative of a constant is 0.




Thus,



Question 2.

Differentiate the following functions with respect to x:

tan2x


Answer:

Let y = tan2x


On differentiating y with respect to x, we get



We know


[using chain rule]



However,




Thus,



Question 3.

Differentiate the following functions with respect to x:

tan(x° + 45°)


Answer:

Let y = tan(x° + 45°)


First, we will convert the angle from degrees to radians.


We have



On differentiating y with respect to x, we get



We know


[using chain rule]




However, and derivative of a constant is 0.




Thus,



Question 4.

Differentiate the following functions with respect to x:

sin(log x)


Answer:

Let y = sin(log x)


On differentiating y with respect to x, we get



We know


[using chain rule]


However,




Thus,



Question 5.

Differentiate the following functions with respect to x:



Answer:

Let


On differentiating y with respect to x, we get



We know


[using chain rule]


We have


[using chain rule]



However,





Thus,



Question 6.

Differentiate the following functions with respect to x:

etan x


Answer:

Let y = etan x


On differentiating y with respect to x, we get



We know


[using chain rule]


We have



Thus,



Question 7.

Differentiate the following functions with respect to x:

sin2(2x + 1)


Answer:

Let y = sin2(2x + 1)


On differentiating y with respect to x, we get



We know


[using chain rule]



We have


[using chain rule]


[∵ sin(2θ) = 2sinθcosθ]




However, and derivative of a constant is 0.




Thus,



Question 8.

Differentiate the following functions with respect to x:

log7(2x – 3)


Answer:

Let y = log7(2x – 3)


Recall that.



On differentiating y with respect to x, we get




We know


[using chain rule]




However, and derivative of a constant is 0.




Thus,



Question 9.

Differentiate the following functions with respect to x:

tan(5x°)


Answer:

Let y = tan(5x°)


First, we will convert the angle from degrees to radians.


We have



On differentiating y with respect to x, we get



We know


[using chain rule]




However,




Thus,



Question 10.

Differentiate the following functions with respect to x:



Answer:

Let


On differentiating y with respect to x, we get



We know


[using chain rule]


We have





Thus,



Question 11.

Differentiate the following functions with respect to x:



Answer:

Let


On differentiating y with respect to x, we get



We know


[using chain rule]


We have




Thus,



Question 12.

Differentiate the following functions with respect to x:

logx3


Answer:

Let y = logx3


Recall that.



On differentiating y with respect to x, we get





We know


[using chain rule]



We have









Thus,



Question 13.

Differentiate the following functions with respect to x:



Answer:

Let


On differentiating y with respect to x, we get



We know


[using chain rule]




We have and





Thus,



Question 14.

Differentiate the following functions with respect to x:



Answer:

Let


On differentiating y with respect to x, we get




We know


[using chain rule]



Recall that (quotient rule)




However, and derivative of a constant is 0.












Thus,



Question 15.

Differentiate the following functions with respect to x:

3x log x


Answer:

Let


On differentiating y with respect to x, we get



We know


[using chain rule]



Recall that (uv)’ = vu’ + uv’ (product rule)



We have and





Thus,



Question 16.

Differentiate the following functions with respect to x:



Answer:

Let


On differentiating y with respect to x, we get




We know


[using chain rule]



Recall that (quotient rule)




We know and derivative of a constant is 0.














(∵ sin2θ + cos2θ = 1)






(∵ sin2θ + cos2θ = 1)






Thus,



Question 17.

Differentiate the following functions with respect to x:



Answer:

Let


On differentiating y with respect to x, we get




We know


[using chain rule]



Recall that (quotient rule)




However, and derivative of a constant is 0.












Thus,



Question 18.

Differentiate the following functions with respect to x:

(log sin x)2


Answer:

Let y = (log sin x)2


On differentiating y with respect to x, we get



We know


[using chain rule]



We have


[using chain rule]



However,





Thus,



Question 19.

Differentiate the following functions with respect to x:



Answer:

Let


On differentiating y with respect to x, we get




We know


[using chain rule]



Recall that (quotient rule)




However, and derivative of a constant is 0.











Thus,



Question 20.

Differentiate the following functions with respect to x:



Answer:

Let


On differentiating y with respect to x, we get



We know


[using chain rule]


Recall that (quotient rule)




However, and derivative of a constant is 0.








Thus,



Question 21.

Differentiate the following functions with respect to x:

e3x cos(2x)


Answer:

Let y = e3xcos(2x)


On differentiating y with respect to x, we get




Recall that (uv)’ = vu’ + uv’ (product rule)



We know and


[chain rule]





We have





Thus,



Question 22.

Differentiate the following functions with respect to x:

sin(log sin x)


Answer:

Let y = sin(log sin x)


On differentiating y with respect to x, we get



We know


[using chain rule]


We have


[using chain rule]



However,





Thus,



Question 23.

Differentiate the following functions with respect to x:

etan 3x


Answer:

Let y = etan 3x


On differentiating y with respect to x, we get



We know


[using chain rule]


We have


[using chain rule]



However,




Thus,



Question 24.

Differentiate the following functions with respect to x:



Answer:

Let


On differentiating y with respect to x, we get



We know


[using chain rule]



We have


[using chain rule]



However,





Thus,



Question 25.

Differentiate the following functions with respect to x:



Answer:

Let



We have sin2θ = 2sinθcosθ and 1 + cos2θ = 2cos2θ.





On differentiating y with respect to x, we get



We know


[using chain rule]



We have




However,





[∵ sin2θ = 2sinθcosθ]




Thus,



Question 26.

Differentiate the following functions with respect to x:



Answer:

Let


On differentiating y with respect to x, we get




We know


[using chain rule]



We know


[using chain rule]





Recall that (quotient rule)




We know and derivative of a constant is 0.








(∵ sin2θ + cos2θ = 1)




Thus,



Question 27.

Differentiate the following functions with respect to x:

tan(esin x)


Answer:

Let y = tan(esin x)


On differentiating y with respect to x, we get



We know


[using chain rule]


We have


[using chain rule]


However,




Thus,



Question 28.

Differentiate the following functions with respect to x:



Answer:

Let


On differentiating y with respect to x, we get



We know


[using chain rule]




We know and


[using chain rule]



However, and derivative of a constant is 0.








Thus,



Question 29.

Differentiate the following functions with respect to x:



Answer:

Let


On differentiating y with respect to x, we get



Recall that (quotient rule)



We have (uv)’ = vu’ + uv’ (product rule)



We know, and









Thus,



Question 30.

Differentiate the following functions with respect to x:

log(cosec x – cot x)


Answer:

Let y = log(cosec x – cot x)


On differentiating y with respect to x, we get



We know


[using chain rule]



We know and







Thus,



Question 31.

Differentiate the following functions with respect to x:



Answer:

Let


On differentiating y with respect to x, we get



Recall that (quotient rule)




We know




However,










Thus,



Question 32.

Differentiate the following functions with respect to x:



Answer:

Let


On differentiating y with respect to x, we get



We know


[using chain rule]



Recall that (quotient rule)




We know, and derivative of constant is 0.














Thus,



Question 33.

Differentiate the following functions with respect to x:

tan–1(ex)


Answer:

Let y = tan–1(ex)


On differentiating y with respect to x, we get



We know


[using chain rule]



However,




Thus,



Question 34.

Differentiate the following functions with respect to x:



Answer:

Let


On differentiating y with respect to x, we get



We know


[using chain rule]


We have


[using chain rule]




However,




Thus,



Question 35.

Differentiate the following functions with respect to x:

sin(2sin–1x)


Answer:

Let y = sin(2sin–1x)


On differentiating y with respect to x, we get



We know


[using chain rule]




We have




Thus,



Question 36.

Differentiate the following functions with respect to x:



Answer:

Let


On differentiating y with respect to x, we get



We know


[using chain rule]


We have


[using chain rule]



However,






Thus,



Question 37.

Differentiate the following functions with respect to x:



Answer:

Let


On differentiating y with respect to x, we get




We know


[using chain rule]



We have


[using chain rule]





However,






Thus,



Question 38.

Differentiate the following functions with respect to x:

log(tan–1x)


Answer:

Let y = log(tan–1x)


On differentiating y with respect to x, we get



We know


[using chain rule]


We have




Thus,



Question 39.

Differentiate the following functions with respect to x:



Answer:

Let


On differentiating y with respect to x, we get



Recall that (quotient rule)



We have (uv)’ = vu’ + uv’ (product rule)



We know, and




However, and derivative of constant is 0.







Thus,



Question 40.

Differentiate the following functions with respect to x:

xsin(2x) + 5x + kk + (tan2x)3


Answer:

Let y = xsin(2x) + 5x + kk + (tan2x)3


On differentiating y with respect to x, we get





Recall that (uv)’ = vu’ + uv’ (product rule)



We know, and


Also, the derivation of a constant is 0.




We have and




Thus,



Question 41.

Differentiate the following functions with respect to x:

log(3x + 2) – x2log(2x – 1)


Answer:

Let y = log(3x + 2) – x2log(2x – 1)


On differentiating y with respect to x, we get





Recall that (uv)’ = vu’ + uv’ (product rule)




We know and





We have and derivative of a constant is 0.






Thus,



Question 42.

Differentiate the following functions with respect to x:



Answer:

Let


On differentiating y with respect to x, we get




Recall that (quotient rule)



We have (uv)’ = vu’ + uv’ (product rule)



We know and




However,







Thus,



Question 43.

Differentiate the following functions with respect to x:

sin2{log(2x + 3)}


Answer:

Let y = sin2{log(2x + 3)}


On differentiating y with respect to x, we get



We know


[chain rule]



We have



As sin(2θ) = 2sinθcosθ, we have




We know






However, and derivative of a constant is 0.





Thus,



Question 44.

Differentiate the following functions with respect to x:

ex log(sin 2x)


Answer:

Let y = ex log(sin 2x)


On differentiating y with respect to x, we get



We have (uv)’ = vu’ + uv’ (product rule)



We know and


[chain rule]



We have





However,





Thus,



Question 45.

Differentiate the following functions with respect to x:



Answer:

Let








On differentiating y with respect to x, we get





We know





We have and derivative of a constant is 0.





Thus,



Question 46.

Differentiate the following functions with respect to x:



Answer:

Let


On differentiating y with respect to x, we get



We know


[using chain rule]




We know and


Also the derivative of a constant is 0.





However, and







Thus,



Question 47.

Differentiate the following functions with respect to x:

(sin–1 x4)4


Answer:

Let y = (sin–1 x4)4


On differentiating y with respect to x, we get



We know


[using chain rule]



We have


[using chain rule]



We have




Thus,



Question 48.

Differentiate the following functions with respect to x:



Answer:

Let


On differentiating y with respect to x, we get



We have


[using chain rule]






Recall that (quotient rule)




We know





We have and derivative of a constant is 0.








Thus,



Question 49.

Differentiate the following functions with respect to x:



Answer:

Let


On differentiating y with respect to x, we get



Recall that (quotient rule)



We have (uv)’ = vu’ + uv’ (product rule)



We know, and




However, and derivative of a constant is 0.







Thus,



Question 50.

Differentiate the following functions with respect to x:

3e–3xlog(1 + x)


Answer:

Let y = 3e–3xlog(1 + x)


On differentiating y with respect to x, we get




We have (uv)’ = vu’ + uv’ (product rule)



We know and




However, and derivative of a constant is 0.






Thus,



Question 51.

Differentiate the following functions with respect to x:



Answer:

Let


On differentiating y with respect to x, we get



Recall that (quotient rule)




We know and derivative of a constant is 0.


[chain rule]




We know








Thus,



Question 52.

Differentiate the following functions with respect to x:



Answer:

Let


On differentiating y with respect to x, we get



Recall that (quotient rule)



We have (uv)’ = vu’ + uv’ (product rule)



We know and




However, and derivative of a constant is 0.












Thus,



Question 53.

Differentiate the following functions with respect to x:



Answer:

Let


On differentiating y with respect to x, we get



We know


[using chain rule]



We have





However, and derivative of a constant is 0.






[∵ sin2θ = 2sinθcosθ]



[∵ sin(90°+θ) = cosθ]



Thus,



Question 54.

Differentiate the following functions with respect to x:

eaxsec(x)tan(2x)


Answer:

Let y = eaxsec(x)tan(2x)


On differentiating y with respect to x, we get




We have (uv)’ = vu’ + uv’ (product rule)




We will use the product rule once again.



We know, and




However,






Thus,



Question 55.

Differentiate the following functions with respect to x:

log(cos x2)


Answer:

Let y = log(cos x2)


On differentiating y with respect to x, we get



We have


[using chain rule]


We know


[using chain rule]




However,




Thus,



Question 56.

Differentiate the following functions with respect to x:

cos(log x)2


Answer:

Let y = cos(log x)2


On differentiating y with respect to x, we get



We have


[using chain rule]


We know


[chain rule]




However,




Thus,



Question 57.

Differentiate the following functions with respect to x:



Answer:

Let


On differentiating y with respect to x, we get




We know


[using chain rule]



We know


[using chain rule]





Recall that (quotient rule)




We know and derivative of a constant is 0.







Thus,



Question 58.

If show that .


Answer:

Given


On differentiating y with respect to x, we get



We know


[using chain rule]




We know




However, and derivative of a constant is 0.








Thus,



Question 59.

If prove that .


Answer:

Given


On differentiating y with respect to x, we get





We know




However, and derivative of a constant is 0.








But,




Thus,



Question 60.

If prove that .


Answer:

Given


On differentiating y with respect to x, we get



Recall that (quotient rule)




However, and derivative of a constant is 0.





On multiplying both sides with x, we get






But,




Thus,



Question 61.

If prove that .


Answer:

Given


On differentiating y with respect to x, we get



We know


[using chain rule]




We know








Thus,



Question 62.

If prove that


Answer:

Given


On differentiating y with respect to x, we get




We know


[using chain rule]



We know


[using chain rule]





Recall that (quotient rule)




We know and derivative of a constant is 0.








(∵ sec2θ – tan2θ = 1)




But, cos2θ + sin2θ = 1 and cos2θ – sin2θ = cos(2θ).




Thus,



Question 63.

If prove that


Answer:

Given


On differentiating y with respect to x, we get





We know











Thus,



Question 64.

If prove that .


Answer:

Given


On differentiating y with respect to x, we get



Recall that (quotient rule)



We have (uv)’ = vu’ + uv’ (product rule)



We know and





However, and derivative of a constant is 0.










But,



Thus,



Question 65.

If prove that .


Answer:

Given


On differentiating y with respect to x, we get



Recall that (quotient rule)




We know







But,



Thus,



Question 66.

If y = (x – 1)log (x – 1) – (x + 1) log (x +1), prove that .


Answer:

Given y = (x – 1)log(x – 1) – (x + 1)log(x + 1)


On differentiating y with respect to x, we get





Recall that (uv)’ = vu’ + uv’ (product rule)




We know and.


Also, the derivative of a constant is 0.






Thus,



Question 67.

If y = ex cos x, prove that .


Answer:

Given y = excos(x)


On differentiating y with respect to x, we get




Recall that (uv)’ = vu’ + uv’ (product rule)



We know and


[chain rule]







We know



However, cos(A)cos(B) – sin(A)sin(B) = cos(A + B)



Thus,



Question 68.

If prove that .


Answer:

Given


We have 1 + cos(2θ) = 2cos2θ and 1 + cos(2θ) = 2sin2θ.





[∵ log(am) = m × log(a)]



On differentiating y with respect to x, we get



We know


[using chain rule]


However,







We have sin(2θ) = 2sinθcosθ





Thus,



Question 69.

If prove that .


Answer:

Given


On differentiating y with respect to x, we get





We have (uv)’ = vu’ + uv’ (product rule)



We know and





However, and derivative of a constant is 0.






Thus,



Question 70.

If prove that .


Answer:

Given


On differentiating y with respect to x, we get




We know


[using chain rule]




However, and derivative of a constant is 0.





But,





Thus,



Question 71.

If y = ex + e–x, prove that .


Answer:

Given y = ex + e–x


On differentiating y with respect to x, we get




We know


[using chain rule]



We have









But, y = ex + e–x



Thus,



Question 72.

If prove that .


Answer:

Given


On differentiating y with respect to x, we get




We know


[using chain rule]




However, and derivative of a constant is 0.





But,





Thus,



Question 73.

If xy = 4, prove that .


Answer:

Given xy = 4



On differentiating y with respect to x, we get





We know





Now, we will evaluate the LHS of the given equation.






However, xy = 4





[∵ xy = 4]


Thus,



Question 74.

If prove that .


Answer:

Let


On differentiating y with respect to x, we get






We have (uv)’ = vu’ + uv’ (product rule)




We know and





However, and derivative of a constant is 0.









Thus,




Exercise 11.3
Question 1.

Differentiate the following functions with respect to x:



Answer:



Now




Using sin2θ + cos2θ = 1 and 2sinθcosθ = sin2θ


= cos–1(2cosθsinθ )


= cos–1(sin2θ)



Considering the limits,









Therefore,






Differentiating w.r.t x,






Question 2.

Differentiate the following functions with respect to x:



Answer:



Now




Using cos2θ = 2cos2θ – 1


y = cos–1(cosθ)


Considering the limits,


–1< x < 1


–1< cos2θ < 1


0 < 2θ < π



Now, y = cos–1(cosθ)


y = θ



Differentiating w.r.t x, we get




Question 3.

Differentiate the following functions with respect to x:



Answer:



Now




Using cos2θ = 1 – 2sin2θ


y = sin–1(sinθ)


Considering the limits,


0 < x < 1


0 < cos2θ < 1




Now, y = sin–1(sinθ)


y = θ



Differentiating w.r.t x, we get




Question 4.

Differentiate the following functions with respect to x:



Answer:



Now



Using sin2θ + cos2θ = 1


y = sin–1(sinθ)


Considering the limits,


0 < x < 1


0 < cos θ < 1



Now, y = sin–1(sinθ)


y = θ


y = cos–1x


Differentiating w.r.t x, we get




Question 5.

Differentiate the following functions with respect to x:



Answer:


Let x = a sinθ


Now



Using sin2θ + cos2θ = 1




y = tan–1(tanθ)


Considering the limits,


–a < x < a


–a < asin θ < a


–1 < sin θ < 1



Now, y = tan–1(tanθ)


y = θ



Differentiating w.r.t x, we get






Question 6.

Differentiate the following functions with respect to x:



Answer:


Let x = a tanθ


Now



Using 1 + tan2θ = sec2θ





y = sin–1(sinθ)


y = θ



Differentiating w.r.t x, we get






Question 7.

Differentiate the following functions with respect to x:

sin–1 (2x2 – 1), 0 < x < 1


Answer:



Now



Using 2cos2θ – 1 = cos2θ


y = sin–1(cos2θ)



Considering the limits,


0 < x < 1


0 < cos θ < 1



0 < 2θ < π


0 > –2θ > –π



Now,





Differentiating w.r.t x, we get






Question 8.

Differentiate the following functions with respect to x:

sin–1 (1 – 2x2), 0 < x < 1


Answer:



Now



Using 1 – 2sin2θ = cos2θ


y = sin–1(cos2θ)



Considering the limits,


0 < x < 1


0 < sin θ < 1



0 < 2θ < π


0 > –2θ > –π



Now,





Differentiating w.r.t x, we get






Question 9.

Differentiate the following functions with respect to x:



Answer:


Let x = a cotθ


Now



Using 1 + cot2θ = cosec2θ





y = cos–1(cosθ)


y = θ



Differentiating w.r.t x, we get






Question 10.

Differentiate the following functions with respect to x:



Answer:


Now




Using sin(A + B) = sinA cosB + cosA sinB



Considering the limits,



Differentiating it w.r.t x,





Question 11.

Differentiate the following functions with respect to x:



Answer:


Now




Using cos(A – B) = cosA cosB + sinA sinB



Considering the limits,




Now,



Differentiating it w.r.t x,




Question 12.

Differentiate the following functions with respect to x:



Answer:


Let x = sinθ


Now



Using sin2θ + cos2θ = 1




Using 2cos2θ = 1 + cos2θ and 2sinθ cosθ = sin2θ




Considering the limits,


–1 < x < 1


–1 < sin θ < 1




Now,





Differentiating w.r.t x, we get





Question 13.

Differentiate the following functions with respect to x:



Answer:


Let x = a sinθ


Now



Using sin2θ + cos2θ = 1




Using 2cos2θ = 1 + cosθ and 2sinθ cosθ = sin2θ




Considering the limits,


–a < x < a


–1 < sin θ < 1




Now,





Differentiating w.r.t x, we get






Question 14.

Differentiate the following functions with respect to x:



Answer:


Let x = sinθ


Now



Using sin2θ + cos2θ = 1



Now




Using sin(A + B) = sinA cosB + cosA sinB



Considering the limits,


–1 < x < 1


–1 < sin θ < 1





Now,





Differentiating w.r.t x, we get





Question 15.

Differentiate the following functions with respect to x:



Answer:


Let x = sinθ


Now



Using sin2θ + cos2θ = 1



Now




Using cos(A – B) = cosA cosB + sinA sinB



Considering the limits,


–1 < x < 1


–1 < sin θ < 1





Now,





Differentiating w.r.t x, we get





Question 16.

Differentiate the following functions with respect to x:



Answer:


Let 2x = tanθ





Considering the limits,



–1 < 2x < 1


–1 < tanθ < 1




Now,


y = tan–1(tan2θ)


y = 2θ


y = 2tan–1(2x)


Differentiating w.r.t x, we get






Question 17.

Differentiate the following functions with respect to x:



Answer:


Let 2x = tanθ






Considering the limits,


–∞ < x < 0


2–∞ < 2x < 20


0 < tanθ < 1




Now,


y = tan–1(tan2θ)


y = 2θ


y = 2tan–1(2x)


Differentiating w.r.t x, we get






Question 18.

Differentiate the following functions with respect to x:



Answer:


Let ax = tanθ





Considering the limits,


–∞ < x < 0


a–∞ < ax < a0


0 < tanθ < 1




Now,


y = tan–1(tan2θ)


y = 2θ


y = 2tan–1(ax)


Differentiating w.r.t x, we get






Question 19.

Differentiate the following functions with respect to x:



Answer:


Let x = cos2θ


Now



Using 1 – 2sin2θ = cos2θ and 2cos2θ – 1 = cos2θ



Now




Using sin(A + B) = sinA cosB + cosA sinB



Considering the limits,


0 < x < 1


0 < cos 2θ < 1




Now,





Differentiating w.r.t x, we get






Question 20.

Differentiate the following functions with respect to x:



Answer:


Let ax = tanθ


Now



Using sec2θ = 1+ tan2θ





Using 2sin2θ = 1 – cos2θ and 2sinθ cosθ = sin2θ






Differentiating w.r.t x, we get






Question 21.

Differentiate the following functions with respect to x:



Answer:


Function y is defined for all real numbers where cosx ≠ –1


Using 2cos2θ = 1 + cos2θ and 2sinθ cosθ = sin2θ





Differentiating w.r.t x, we get





Question 22.

Differentiate the following functions with respect to x:



Answer:


Let x = cotθ


Now



Using, 1 + cot2θ = cosec2θ


Now




y = sin–1(sin θ)


y = θ


y =cot–1x


Differentiating w.r.t x we get





Question 23.

Differentiate the following functions with respect to x:



Answer:


Let xn = tanθ


Now





Considering the limits,


0 < x < ∞


0 < xn < ∞



Now,


y = cos–1(cos2θ)


y = 2θ


y = tan–1(xn)


Differentiating w.r.t x, we get






Question 24.

Differentiate the following functions with respect to x:



Answer:






Differentiating w.r.t x we get





Question 25.

Differentiate the following functions with respect to x:



Answer:




Differentiating w.r.t x we get






Question 26.

Differentiate the following functions with respect to x:



Answer:




Differentiating w.r.t x we get






Question 27.

Differentiate the following functions with respect to x:



Answer:


Dividing numerator and denominator by b







Differentiating w.r.t x we get






Question 28.

Differentiate the following functions with respect to x:



Answer:


Dividing numerator and denominator by b





Differentiating w.r.t x we get






Question 29.

Differentiate the following functions with respect to x:



Answer:


Dividing numerator and denominator by x





Differentiating w.r.t x we get







Question 30.

Differentiate the following functions with respect to x:



Answer:


Arranging the terms in equation





Differentiating w.r.t x we get






Question 31.

Differentiate the following functions with respect to x:



Answer:


Arranging the terms in equation





Differentiating w.r.t x we get






Question 32.

Differentiate the following functions with respect to x:



Answer:


Dividing numerator and denominator by cosx








Differentiating w.r.t x we get






Question 33.

Differentiate the following functions with respect to x:



Answer:


Arranging the terms in equation





Differentiating w.r.t x we get







Question 34.

Differentiate the following functions with respect to x:



Answer:


For function to be defined



Since the quantity is positive always





This condition is always true, hence function is always defined.



Let 2x = tanθ




Now,


y = sin–1(sin2θ)


y = 2θ


y = 2tan–1(2x)


Differentiating w.r.t x, we get






Question 35.

If 0 < x < 1, prove that


Answer:


Put x = tan θ







Considering the limits


0 < x < 1


0 < tan θ < 1




Now,


y = 2θ + 2θ


y = 4θ


y = 4tan–1x


Differentiating w.r.t x we get





Question 36.

If 0 < x < ∞, prove that


Answer:


Put x = tan θ







Considering the limits


0 < x < ∞


0 < tan θ < ∞



Now,


y = θ + θ


y = 2θ


y = 2tan–1x


Differentiating w.r.t x we get





Question 37.

Differentiate the following with respect to x:

cos–1 (sin x)


Answer:

y = cos–1(sinx)


Function is defined for all x




Differentiating w.r.t x we get





Question 38.

Differentiate the following with respect to x:



Answer:


Put x = tan θ









Differentiating w.r.t x we get






Question 39.

If show that is independent of x.


Answer:


Multiplying numerator and denominator







Using sin2θ + cos2θ = 1



Using 2sinθ cosθ = sin2θ and 2cos2θ – 1 = cos2θ



Now




Differentiating w.r.t x, we get





Question 40.

If x > 0, prove that


Answer:




Put x = tan θ





Considering the limits


0 < x < ∞


0 < tan θ < ∞




Now,


y = 2θ + 2θ


y = 4θ


y = 4tan–1x


Differentiating w.r.t x we get





Question 41.

If x > 0. Find .


Answer:






Now differentiating w.r.t x we get





Question 42.

If find .


Answer:


Put x =cos 2θ





y = sin[2tan–1(tan θ)]


y = sin(2θ)






Differentiating w.r.t x we get







Question 43.

If find .


Answer:


Put 2x = cos θ



y = cos–1(cosθ) + 2cos–1(sinθ )



Considering the limits



0 < 2x < 1


0 < cosθ < 1





Now,





y = π – cos–1(2x)


Differentiating w.r.t x we get






Question 44.

If the derivative of tan–1 (a + bx) takes the value 1 at x = 0, prove that 1 + a2 = b.


Answer:

y = tan–1(a + bx)


And y’(0) = 1


Now




At x = 0,




⇒ b = 1 + a2



Question 45.

If y = cos–1 (2x) + 2 cos–1 < x < 0, find .


Answer:


Put 2x = cos θ



y = cos–1(cosθ) + 2cos–1(sinθ )



Considering the limits



–1 < 2x < 0


–1 < cosθ < 0





Now,





y = –π + cos–1(2x)


Differentiating w.r.t x we get






Question 46.

If find .


Answer:


Put x = cos 2θ






Dividing by cosθ both numerator and denominator,








Differentiating w.r.t x, we get






Question 47.

If find .


Answer:


Put x = cos θ





Now,






Again,







Differentiating w.r.t x, we get






Question 48.

Differentiate with respect to x.


Answer:




Put 6x = tanθ




Now,


y = sin–1(sin2θ)


y = 2θ


y = 2tan–1(6x)


Differentiating w.r.t x, we get






Question 49.

If then find .


Answer:





Using sin2θ + cos2θ = 1



Using 2sinθcosθ = sin2θ


y = sin–1(sin2θ)


Considering the limits,







For



Now, y = sin–1(sin2θ)


y = 2θ


y = 2cos–1x


Differentiating w.r.t x, we get



For



Now, y = sin–1(sin2θ)


y = –2θ


y = –2cos–1x


Differentiating w.r.t x, we get





Exercise 11.4
Question 1.

Find in each of the following:

xy = c2


Answer:

We are given with an equation xy = c2; we have to find of it, so by differentiating the equation on both sides with respect to x, we get,


By using the product rule on the left hand side,



x + y(1) = 0



Or we can further solve it by putting the value of y,




Question 2.

Find in each of the following:

y3 – 3xy2 = x3 + 3x2 y


Answer:

We are given with an equation y3 – 3xy2 = x3 + 3x2y, we have to find of it, so by differentiating the equation on both sides with respect to x, we get,


3y2 ‐ 3[y2(1) + 2xy ] = 3x2 + 3[2xy + x2]


Taking terms to left hand side and taking common , we get,


[3y2 ‐ 6xy ‐ 3x2] = 3x2 + 6xy + 3y2




Question 3.

Find in each of the following:

x2/3 + y2/3 = a2/3


Answer:

We are given with an equation , we have to find of it, so by differentiating the equation on both sides with respect to x, we get,




Or we can write it as,




Question 4.

Find in each of the following:

4x + 3y = log (4x – 3y)


Answer:

We are given with an equation 4x + 3y = log(4x – 3y), we have to find of it, so by differentiating the equation on both sides with respect to x, we get,


4 + 3[4 – 3]


3





Question 5.

Find in each of the following:



Answer:

We are given with an equation , we have to find of it, so by differentiating the equation on both sides with respect to x, we get,





Question 6.

Find in each of the following:

x5 + y5 = 5xy


Answer:

We are given with an equation x5 + y5 = 5xy, we have to find of it, so by differentiating the equation on both sides with respect to x, we get,


5x4 + 5y4 = 5[y(1) + x]


[y4 – x] = y – x4




Question 7.

Find in each of the following:

(x + y)2 = 2axy


Answer:

We are given with an equation (x + y)2 = 2axy, we have to find of it, so by differentiating the equation on both sides with respect to x, we get,


2(x + y)(1 + ) = 2a[y + x]


x + y + [x + y] = a[y + x]


[x + y – ax] = ay – x – y




Question 8.

Find in each of the following:

(x2 + y2)2 = xy


Answer:

We are given with an equation (x2 + y2)2 = xy, we have to find of it, so by differentiating the equation on both sides with respect to x, we get,


2(x2 + y2)[2x + 2y] = y(1) + x


[4y(x2 + y2) – x] = y – 4x(x2 + y2)




Question 9.

Find in each of the following:

tan – 1 (x2 + y2) = a


Answer:

We are given with an equation tan – 1(x2 + y2) = a, we have to find of it, so by differentiating the equation on both sides with respect to x, we get,





Question 10.

Find in each of the following:



Answer:

We are given with an equation ex – y = log() = logx – logy, we have to find of it, so by differentiating the equation on both sides with respect to x, we get,


ex – y(1 – ) =


[ – ex – y] = – ex – y





Question 11.

Find in each of the following:

sinxy + cos (x + y) = 1


Answer:

We are given with an equation sinxy + cos(x + y) = 1, we have to find of it, so by differentiating the equation on both sides with respect to x, we get,


cosxy (y + x) – sin(x + y) (1 + ) = 0


[x cosxy – sin(x + y)] = sin(x + y) – y cosxy




Question 12.

If prove that


Answer:

We are given with an equation = a(x – y), we have to prove that by using the given equation we will first find the value of and we will put this in the equation we have to prove, so by differentiating the equation on both sides with respect to x, we get,


Put x = sinA and y = sinB in the given equation,


= a(sinA – sinB)


cosA + cosB = a(sinA – sinB)


2cos()cos() = a2cos()sin()


By using cosA + cosB = 2cos()cos() and sinA – sinB = 2cos()sin()


a = cot()


cot – 1a =


2cot – 1a = A – B


2cot – 1a = sin – 1x – sin – 1y


0 =




Question 13.

If prove that


Answer:

We are given with an equation = 1, we have to prove that by using the given equation we will first find the value of and we will put this in the equation we have to prove, so by differentiating the equation on both sides with respect to x, we get,


Put x = sinA and y = sinB in the given equation,


= 1


sinB cosA + sinA cosB = 1


sin(A + B) = 1


sin – 11 = A + B


= sin – 1x + sin – 1y


Differentiating we get,


0 =




Question 14.

If xy = 1, prove that


Answer:

We are given with an equation xy = 1, we have to prove that + y2 = 0 by using the given equation we will first find the value of and we will put this in the equation we have to prove, so by differentiating the equation on both sides with respect to x, we get,


By using product rule, we get,


y(1) + x = 0



Or we can further solve it by using the given equation,



By putting this value in the L.H.S. of the equation, we get,


–y2 + y2 = 0 = R.H.S.



Question 15.

If xy2 = 1, prove that


Answer:

We are given with an equation xy2 = 1, we have to prove that 2 + y3 = 0 by using the given equation we will first find the value of and we will put this in the equation we have to prove, so by differentiating the equation on both sides with respect to x, we get,


y2(1) + 2xy = 0



Or we can further solve it by using the given equation,




By putting this value in the L.H.S. of the equation, we get,


2() + y3 = 0 = R.H.S.



Question 16.

If prove that


Answer:

We are given with an equation xy2 = 1, we have to prove that 2 + y3 = 0 by using the given equation we will first find the value of and we will put this in the equation we have to prove


But first we need to simplify this equation in accordance with our result, which is that in our result there is no square root and our derivative is only in the form of x.


= 0



Squaring both sides,


x2(1 + y) = y2(1 + x)


x2 + x2y = y2 + xy2


x2 – y2 = xy2 – x2y


(x – y)(x + y) = xy(y – x)


x + y = – xy


y =


So, now by differentiating the equation on both sides with respect to x, we get,


By using quotient rule, we get,





Question 17.

If prove that


Answer:

We are given with an equation logtan – 1(), we have to prove that by using the given equation we will first find the value of and we will put this in the equation we have to prove, so by differentiating the equation on both sides with respect to x, we get,


log(x2 + y2) = 2tan – 1()



x + y = x – y




Question 18.

If prove that


Answer:

We are given with an equation sec() = a, we have to prove that by using the given equation we will first find the value of and we will put this in the equation we have to prove, so by differentiating the equation on both sides with respect to x, we get,


sec() tan() [] = 0


[] = 0


– 2y + 2x = 0




Question 19.

If prove that


Answer:

We are given with an equation tan – 1() = a, we have to prove that by using the given equation we will first find the value of and we will put this in the equation we have to prove, so by differentiating the equation on both sides with respect to x, we get,


= tan a


x2 – y2 = (x2 + y2)tan a


Now differentiating with respect to x, we get,


2x – 2y = (2x + 2y)tan a


[ytan a + y] = x – xtanx





Question 20.

If xy log (x + y) = 1, prove that


Answer:

We are given with an equation xy log(x + y) = 1, we have to prove that by using the given equation we will first find the value of and we will put this in the equation we have to prove, so by differentiating the equation on both sides with respect to x, we get,


By using the triple product rule, which is, ,


(1)y log(x + y) + x log(x + y) + xy = 0


From the equation put log(x + y) =


= 0


= 0


= 0




Question 21.

If y = x sin (a + y), prove that


Answer:

We are given with an equation y = x sin(a + y), we have to prove that by using the given equation we will first find the value of and we will put this in the equation we have to prove, so by differentiating the equation on both sides with respect to x, we get,


= (1) sin(a + y) + x cos(a + y)



We can further solve it by using the given equation,





Question 22.

If x sin (a + y) + sin a cos (a + y) = 0, prove that


Answer:

We are given with an equation x sin(a + y) + sina cos(a + y) = 0 , we have to prove that by using the given equation we will first find the value of and we will put this in the equation we have to prove, so by differentiating the equation on both sides with respect to x, we get,


tan(a + y) =


sec2(a + y)


we can further solve it by using the given equation,


sec2(a + y)




Question 23.

If y – x sin y, prove that


Answer:

We are given with an equation y = x siny , we have to prove that by using the given equation we will first find the value of and we will put this in the equation we have to prove, so by differentiating the equation on both sides with respect to x, we get,


= siny + x cosy


[1 – x cosy] = siny




Question 24.

If show that


Answer:

We are given with an equation ylog() , we have to prove that


(x2 + 1) + xy + 1 = 0 by using the given equation we will first find the value of and we will put this in the equation we have to prove, so by differentiating the equation on both sides with respect to x, we get,





= –1


xy + (x2 + 1) + 1 = 0



Question 25.

If find .


Answer:

We are given with an equation sin(xy) + = x2 – y2 , we have to find by using the given equation, so by differentiating the equation on both sides with respect to x, we get,


cos(xy) [(1)y + x] + = 2x – 2y


ycos(xy) + xcos(xy) = 2x – 2y


[x cos(xy) + + 2y] = 2x – y cos(xy) +





Question 26.

If tan (x + y) + tan (x – y) = 1, find.


Answer:

We are given with an equation tan(x + y) + tan(x – y) = 1 , we have to find by using the given equation, so by differentiating the equation on both sides with respect to x, we get,


sec2(x + y)[1 + ] + sec2(x – y)[1 – ] = 0


[sec2(x + y) – sec2(x – y)] + sec2(x + y) + sec2(x – y) = 0




Question 27.

If ex + ey = ex + y, prove that or,


Answer:

We are given with an equation ex + ey = ex + y, we have to prove that by using the given equation we will first find the value of and we will put this in the equation we have to prove, so by differentiating the equation on both sides with respect to x, we get,


ex + ey = e(x + y) [1 + ]


[ey – ex + y] = ex + y – ex





Question 28.

If cos y = x cos(a + y), with cos a ≠±1, prove that


Answer:

We are given with an equation cosy = x cos(a + y) , we have to prove that by using the given equation we will first find the value of and we will put this in the equation we have to prove, so by differentiating the equation on both sides with respect to x, we get,


– siny = cos(a + y) – x sin(a + y)


[xsin(a + y) – siny] = cos(a + y)



We can further solve it by using the given equation,




By using sinA cosB – cosA sinB = sin(A – B)




Question 29.

If sin2y + cosxy = k, find at x = 1,


Answer:

We are given with an equation sin2y + cos(xy) = k , we have to find at x = 1, y = by using the given equation, so by differentiating the equation on both sides with respect to x, we get,


2siny cosy – sin(xy)[(1)y + x] = 0


[2siny cosy – xsin(xy)] = ysin(xy)



By putting the value of point in the derivative, which is x = 1, y = ,


(x = 1,y =π/4) =


(x = 1,y =π/4) =



Question 30.

If find at


Answer:

We are given with an equation y = {logcosxsinx} {logsinxcosx} – 1 + sin – 1(), we have to find at


x = by using the given equation, so by differentiating the equation on both sides with respect to x, we get,


By using the properties of logarithms,


y = {logcosxsinx}2 + sin – 1()


y = {}2 + sin – 1()


= 2{}


= 2{}


= 2{}


Now putting the value of x = in the derivative solved above, we get,


(x =π/4) = 2{1} +


(x =π/4) = 2{1} +


(x =π/4) = 2{1} +


(x =π/4) = +



Question 31.

If show that


Answer:

We are given with an equation = c , we have to prove that by using the given equation we will first find the value of and we will put this in the equation we have to prove, so by differentiating the equation on both sides with respect to x, we get,


= 0


= 0


= 0


[ ] =



×










Exercise 11.5
Question 1.

Differentiate the following functions with respect to x :

x1/x


Answer:


Taking log both the sides:




{log xa = alog x}


Differentiating with respect to x:













Question 2.

Differentiate the following functions with respect to x :

xsin x


Answer:

Let y = xsin x


Taking log both the sides:


log y = log (xsin x )


log y = sin x log x {log xa = alog x}


Differentiating with respect to x:









Put the value of y = xsinx :




Question 3.

Differentiate the following functions with respect to x :

(1 + cos x)x


Answer:

Let y = (1 + cos x)x


Taking log both the sides:


⇒ log y = log (1 + cos x)x


⇒ log y = x log (1+ cos x) {log xa = alog x}


Differentiating with respect to x:












Put the value of y = (1 + cos x)x :




Question 4.

Differentiate the following functions with respect to x :



Answer:


Taking log both the sides:



{log xa = alog x}


Differentiating with respect to x:









:




Question 5.

Differentiate the following functions with respect to x :

(log x)x


Answer:

Let y = (log x)x


Taking log both the sides:


⇒ log y = log (log x)x


⇒ log y = x log (log x) {log xa = alog x}


Differentiating with respect to x:









Put (log x)x




Question 6.

Differentiate the following functions with respect to x :

(log x)cos x


Answer:

Let y = (log x)cos x


Taking log both the sides:


⇒ log y = log (log x)cos x


⇒ log y = cos x log log x {log xa = alog x}


Differentiating with respect to x:









Put (log x)cos x:




Question 7.

Differentiate the following functions with respect to x :

(sin x)cos x


Answer:

Let y = (sin x)cos x


Taking log both the sides:


⇒ log y = log (sin x)cos x


⇒ log y = cos x log sin x {log xa = alog x}


Differentiating with respect to x:









Put (sin x)cos x :




Question 8.

Differentiate the following functions with respect to x :

ex logx


Answer:

Let y = ex log x


Taking log both the sides:


⇒ log y = log (e)x log x


⇒ log y = x log x log e {log xa = alog x}


⇒ log y = x log x {log e = 1}


Differentiating with respect to x:









Put ex logx :



{ elog a = a; alog x = xa}




Question 9.

Differentiate the following functions with respect to x :

(sin x)log x


Answer:

Let y = (sin x)log x


Taking log both the sides:


⇒ log y = log (sin x)log x


⇒ log y = log x log sin x {log xa = alog x}


Differentiating with respect to x:









Put (sin x)log x :




Question 10.

Differentiate the following functions with respect to x :

10log sin x


Answer:

Let y = 10log sin x


Taking log both the sides:


⇒ log y = log 10log sin x


⇒ log y = log sin x log 10 {log xa = alog x}


Differentiating with respect to x:









Put 10log sin x:




Question 11.

Differentiate the following functions with respect to x :

(log x)log x


Answer:

Let y = (log x)log x


Taking log both the sides:


⇒ log y = log (log x)log x


⇒ log y = log x log (log x) {log xa = alog x}


Differentiating with respect to x:










Put (log x)log x :




Question 12.

Differentiate the following functions with respect to x :

10(10x)


Answer:

Let y = 1010x


Taking log both the sides:


⇒ log y = log 1010x


⇒ log y = 10x log 10 {log xa = alog x}


⇒ log y = (10log 10)x


Differentiating with respect to x:



{Here 10log (10) is a constant term}







Put 1010 x:




Question 13.

Differentiate the following functions with respect to x :

sin (xx)


Answer:

Let y = sin (xx)


Take sin inverse both sides:


⇒ sin-1 y = sin-1 (sin xx)


⇒ sin-1 y = xx


Taking log both the sides:


⇒ log (sin-1 y) = log xx


⇒ log (sin-1 y) = x log x {log xa = alog x}


Differentiating with respect to x:











Put sin (xx) :




{sin2 x + cos2 x=1}




Question 14.

Differentiate the following functions with respect to x :

(sin-1 x)x


Answer:

Let y = (sin-1 x)x


Taking log both the sides:


⇒ log y = log (sin-1 x)x


⇒ log y = x log (sin-1 x) {log xa = alog x}


Differentiating with respect to x:











Put (sin-1 x)x:




Question 15.

Differentiate the following functions with respect to x :



Answer:


Taking log both the sides:



⇒ log y = sin-1 x log x{log xa = alog x}


Differentiating with respect to x:












Question 16.

Differentiate the following functions with respect to x :

(tan x)1/x


Answer:


Taking log both the sides:



{log xa = alog x}


Differentiating with respect to x:













Question 17.

Differentiate the following functions with respect to x :



Answer:


Taking log both the sides:



⇒ log y = tan-1 x log x{log xa = alog x}


Differentiating with respect to x:












Question 18.

Differentiate the following functions with respect to x :



Answer:


Taking log both the sides:



{log (ab) = log a +log b}




{log xa = alog x}



Differentiating with respect to x:














Question 19.

Differentiate the following functions with respect to x :



Answer:


⇒ y = a + b






Taking log both the sides:




{log xa = alog x}


Differentiating with respect to x:
























Question 20.

Differentiate the following functions with respect to x :



Answer:


⇒ y = a + b






Taking log both the sides:




{log xa = alog x}


Differentiating with respect to x:


























Question 21.

Differentiate the following functions with respect to x :

(x cos x)x + (x sin x)1/x


Answer:


⇒ y = a + b






Taking log both the sides:




{log xa = alog x}


Differentiating with respect to x:
















Taking log both the sides:



{log xa = alog x}


Differentiating with respect to x:

















Question 22.

Differentiate the following functions with respect to x :



Answer:


⇒ y = a + b






Taking log both the sides:




{log xa = alog x}


Differentiating with respect to x:

















Taking log both the sides:



{log xa = alog x}


Differentiating with respect to x:
















Question 23.

Differentiate the following functions with respect to x :

esin x + (tan x)x


Answer:

let y = esin x + (tan x)x


⇒ y = a + b


where a= esin x ; b = (tan x)x




a= esin x


Taking log both the sides:


⇒ log a= log esin x


⇒ log a= sin x log e


{log xa = alog x}


⇒ log a= sin x {log e =1}


Differentiating with respect to x:







Put the value of a = esinx



b = (tan x)x


Taking log both the sides:


⇒ log b= log (tan x)x


⇒ log b= x log (tan x)


{log xa = alog x}


Differentiating with respect to x:











Put the value of b = (tan x)x :






Question 24.

Differentiate the following functions with respect to x :

(cos x)x + (sin x)1/x


Answer:


⇒ y = a + b






Taking log both the sides:




{log xa = alog x}


Differentiating with respect to x:














Taking log both the sides:



{log xa = alog x}


Differentiating with respect to x:
















Question 25.

Differentiate the following functions with respect to x :



Answer:


⇒ y = a + b






Taking log both the sides:




{log xa = alog x}


Differentiating with respect to x:












Taking log both the sides:



{log xa = alog x}


Differentiating with respect to x:















Question 26.

Find , when

1y = ex + 10x + xx


Answer:

let y = ex + 10x + xx


⇒ y = a + b + c


where a= ex; b = 10x ; c = xx




a= ex


Taking log both the sides:


⇒ log a= log ex


⇒ log a= x log e


{log xa = alog x}


⇒ log a= x {log e =1}


Differentiating with respect to x:







Put the value of a = ex



b = 10x


Taking log both the sides:


⇒ log b= log 10x


⇒ log b= x log 10


{log xa = alog x}


Differentiating with respect to x:









Put the value of b = 10x



c = xx


Taking log both the sides:


⇒ log c= log xx


⇒ log c= x log x


{log xa = alog x}


Differentiating with respect to x:









Put the value of c = xx






Question 27.

Find , when

y = xn + nx + xx + nn


Answer:

let y = xn + nx + xx + nn


⇒ y = a + b + c + m


where a= xn; b = nx ; c = xx ; m= nn




a= xn


Taking log both the sides:


⇒ log a= log xn


⇒ log a= n log x


{log xa = alog x}


⇒ log a= n log x {log e =1}


Differentiating with respect to x:










Put the value of a = xn





b = nx


Taking log both the sides:


⇒ log b= log nx


⇒ log b= x log n


{log xa = alog x}


Differentiating with respect to x:









Put the value of b = nx:



c = xx


Taking log both the sides:


⇒ log c= log xx


⇒ log c= x log x


{log xa = alog x}


Differentiating with respect to x:









Put the value of c = xx



m = nn









Question 28.

Find , when



Answer:




Take log both sides:






{log xa = alog x}


Differentiating with respect to x:














Question 29.

Find , when



Answer:



Take log both sides:





{log xa = alog x}


{log e = 1}


Differentiating with respect to x:
















Question 30.

Find , when

y = e3x sin 4x 2x


Answer:

Let y = e3x sin 4x 2x


Take log both sides:


⇒ log y = log (e3x sin 4x 2x)


⇒ log y = log (e3x ) + log (sin 4x) + log (2x)



⇒ log y = 3x log e+ log (sin 4x) + x log 2 {log xa = alog x}


⇒ log y = 3x + log (sin 4x) + x log 2 {log e = 1}


Differentiating with respect to x:













Question 31.

Find , when

y = sin x sin 2x sin 3x sin 4x


Answer:

Let y = sin x sin 2x sin 3x sin 4x


Take log both sides:


⇒ log y = log (sin x sin 2x sin 3x sin 4x)


⇒ log y = log (sin x ) + log (sin 2x) + log (sin 3x) + log (sin 4x)



Differentiating with respect to x:















Question 32.

Find , when

y = xsin x + (sin x)x


Answer:

let y = xsin x + (sin x)x


⇒ y = a + b


where a= xsin x; b = (sin x)x




a= xsin x


Taking log both the sides:


⇒ log a= log xsin x


⇒ log a= sin x log x


{log xa = alog x}


Differentiating with respect to x:









Put the value of a = xsinx :



b = (sin x)x


Taking log both the sides:


⇒ log b= log (sin x)x


⇒ log b= x log (sin x)


{log xa = alog x}


Differentiating with respect to x:











Put the value of b = (sin x)x :






Question 33.

Find , when

y = (sin x)cos x + (cos x)sin x


Answer:

let y = (sin x)cos x + (cos x)sin x


⇒ y = a + b


where a= (sin x)cos x; b = (cos x)sin x




a= (sin x)cos x


Taking log both the sides:


⇒ log a= log (sin x)cos x


⇒ log a= cos x log (sin x)


{log xa = alog x}


Differentiating with respect to x:











b = (cos x)sin x


Taking log both the sides:


⇒ log b= log (cos x)sin x


⇒ log b= sin x log (cos x)


{log xa = alog x}


Differentiating with respect to x:














Question 34.

Find , when

y = (tan x)cot x + (cot x)tan x


Answer:

let y = (tan x)cot x + (cot x)tan x


⇒ y = a + b


where a= (tan x)cot x ; b = (cot x)tan x




a= (tan x)cot x


Taking log both the sides:


⇒ log a= log (tan x)cot x


⇒ log a= cot x log (tan x)


{log xa = alog x}


Differentiating with respect to x:












b = (cot x)tan x


Taking log both the sides:


⇒ log b= log (cot x)tan x


⇒ log b= tan x log (cot x)


{log xa = alog x}


Differentiating with respect to x:















Question 35.

Find , when



Answer:


⇒ y = a + b





a = (sin x)x


Taking log both the sides:


⇒ log a= log (sin x)x


⇒ log a= x log (sin x)


{log xa = alog x}


Differentiating with respect to x:











Put the value of a= (sin x)x :





Differentiating with respect to x:














Question 36.

Find , when

y = xcos x + (sin x)tan x


Answer:

let y = xcos x + (sin x)tan x


⇒ y = a + b


where a= xcos x ; b = (sin x)tan x




a= xcos x


Taking log both the sides:


⇒ log a= log (x)cos x


⇒ log a= cos x log x


{log xa = alog x}


Differentiating with respect to x:











b = (sin x)tan x


Taking log both the sides:


⇒ log b= log (sin x)tan x


⇒ log b= tan x log (sin x)


{log xa = alog x}


Differentiating with respect to x:














Question 37.

Find , when

y = xx + (sin x)x


Answer:

let y = x x + (sin x) x


⇒ y = a + b


where a= x x ; b = (sin x) x




a= xx


Taking log both the sides:


⇒ log a= log (x)x


⇒ log a= x log x


{log xa = alog x}


Differentiating with respect to x:











b = (sin x)x


Taking log both the sides:


⇒ log b= log (sin x)x


⇒ log b= x log (sin x)


{log xa = alog x}


Differentiating with respect to x:















Question 38.

Find , when

y = (tan x)log x + cos2


Answer:


⇒ y = a + b





a= (tan x)log x


Taking log both the sides:


⇒ log a= log (tan x)log x


⇒ log a= log x . log (tan x)


{log xa = alog x}


Differentiating with respect to x:















Differentiating with respect to x:










Question 39.

Find , when

y = xx + x1/x


Answer:

Here,


y = xx + x1/x


=


y =


[ Since]


Differentiating it with respect to x using the chain rule and product rule,









Question 40.

Find , when

y = xlog x + (log x)x


Answer:

Here,


y = xlog x + (log x)x


Let


, and


y=u+v


……(i)





Differentiating both sides with respect to x, we get













Therefore from (i), (ii), (iii), we get




Question 41.

If x13 y7 = (x + y)20, prove that


Answer:

Here,


x13 y7 = (x + y)20


Taking log on both sides,



13 log x+7log y = 20 log(x+y)


[ Since, log (AB)=logA+logB ; log=b log a]


Differentiating it with respect to x using the chain rule,











Hence, Proved.



Question 42.

If x16 y9 = (x2 + y)17, prove that


Answer:

Here,


x16 y9 = (x2 + y)17


Taking log on both sides,



16 log x + 9 log y = 17 log(+y)


[ Since, log (AB)=logA+logB ; log =b log a]


Differentiating it with respect to x using the chain rule,














Hence, Proved.



Question 43.

If y = sin (xx), prove that


Answer:

Here,


y = sin (xx) ……(i)


Let ……(ii)


Taking log on both sides,


log u = log


log u = x log x


Differentiating both sides with respect to x,







[from (ii)]


Now, using equation (ii) in (i)


y = sin u


Differentiating both sides with respect to x,




Using equation (ii) and (iii),



Hence Proved.



Question 44.

If xx + yx = 1, prove that


Answer:

Here


xx + yx = 1




[ Since , ]


Differentiating it with respect to x using chain rule and product rule,










Hence Proved.



Question 45.

If xx + yx = 1, find


Answer:

Let xx = u and yx = v


Taking log on both sides we get,


x log x = log u ……(1),


x log y = log v ……(2)


Using


Differentiating both sides of equation (1) we get,




Differentiating both sides of equation (2) we get,




We know that, from question,


u + v = 1


Differentiating both sides we get,



Putinng the value of eq(4) and eq(5) in equation above we get,






Question 46.

If xy + yx = (x + y)x + y, find


Answer:

Here,





Differentiating it with respect to x using chain rule, product rule,











Question 47.

If xmyn = 1, prove that


Answer:

Here,



Taking log on both sides,


log() = log 1


m logx + n logy=log 1 [ Since, log (AB)=logA+logB ; log =b log a]


Differentiating with respect to x







Hence Proved.



Question 48.

If yx = ey – x prove that


Answer:

Here,


Taking log on both sides,



[ Since, log (AB)=logA+logB ; log =b log a]


……(i)


Differentiating with respect to x using product rule,










Hence Proved.



Question 49.

If (sin x)y = (cos y)x, prove that


Answer:

Here,



Taking log on both sides,



[Using log =b log a]


Differentiating it with respect to x using product rule and chain rule,










Hence Proved.



Question 50.

If (cos x)y = (tan y)x, prove that


Answer:

Here,



Taking log on both sides,



[Using log =b log a]


Differentiating it with respect to x using product rule and chain rule,









Question 51.

If ex + ey = ex + y, prove that


Answer:

Here,


……(i)


Differentiating both the sides using chain rule,













Hence Proved.



Question 52.

If ey = yx, prove that


Answer:

Here



Taking log on both sides,




[Using log =b log a]


--------- (i)


Differentiating it with respect to x using product rule,












Hence Proved.



Question 53.

If ex + y – x = 0, prove that


Answer:

Here,


ex + y – x = 0


…… (i)


Differentiating it with respect to x using chain rule,








Hence Proved.



Question 54.

If y = x sin(a + y), prove that


Answer:

Here


y = x sin(a+y)


Differentiating it with respect to x using the chain rule and product rule,






[ Since, ]




Hence Proved.



Question 55.

If x sin (a + y) + sin a cos (a + y) = 0, prove that


Answer:

Here, x sin(a+y) + sin a cos(a+y) = 0


x=


Differentiating it with respect to x using the chain rule and product rule,







[ Since x=]





[ Since a + a = 1]


Hence Proved.



Question 56.

If (sin x)y = x + y, prove that


Answer:

Here


(sin x)y = x + y


Taking log both sides,


log (sin x)y = log(x + y)


y log(sinx)=log(x+y) [Using log =b log a]


Differentiating it with respect to x using the chain rule and product rule,










Hence Proved.



Question 57.

If xy log(x + y) = 1, prove that


Answer:

Here,


xy log(x + y) = 1 ……(i)


Differentiating it with respect to x using the chain rule and product rule,





[Using (i)]






Hence Proved.



Question 58.

If y = x sin y, prove that


Answer:

Here,


y = x sin y


……(i)


Differentiating it with respect to x using product rule,








[From (i)]


Hence Proved.



Question 59.

Find the derivative of the function f(x) given by

f(x) = (1 + x)(1 + x2)(1 + x4)(1 + x8) and hence find f’(1)


Answer:

Here,


f(x) = (1 + x)(1 + x2)(1 + x4)(1 + x8)


f(1) = (2)(2)(2)(2) = 16


Taking log on both sides we get,


Log (f (x)) = log (1 + x) + log (1 + x2) + log (1 + x4) + log(1 + x8)


Differentiating it with respect to x we get,







F’(1) = 120



Question 60.

If find .


Answer:

Here,


Differentiating it with respect to x using chain and quotient rule,










Hence,




Question 61.

If find .


Answer:

Here, y = ……(i)


Taking log on both sides,


log y = log


log y = log


Differentiating it with respect to x using product rule, chain rule,







Using (i),




Question 62.

If xy = ex – y, find .


Answer:

The given function is xy = ex – y


Taking log on both sides, we obtain


log (xy)= log


log x + log y = (x-y) log e


log x + log y = (x-y)1


log x + log y = x-y


Differentiating both sides with respect to x, we obtain








Question 63.

If yx + xy + xx = ab, find .


Answer:

Given that, yx + xy + xx = ab


Putting, u=yx, v=xy, w=xx ,we get


u+v+w=ab


Therefore, ……(i)


Now, u=yx,


Taking log on both sides, we have


log u = x log y


Differentiating both sides with respect to x, we have




So,


……(ii)


Also, v=,


Taking log on both sides, we have


log v = y log x


Differentiating both sides with respect to x, we have




So,


……(iii)


Again, w=,


Taking log on both sides, we have


log w = x log x


Differentiating both sides with respect to x, we have




So,


……(iv)


From (i), (ii), (iii), (iv)





Therefore,




Question 64.

If (cos x)y = (cos y)x find .


Answer:

Here,


Taking log on both sides,




Differentiating it with respect to x using the chain rule and product rule,









Question 65.

If cos y = x cos (a + y), where prove that


Answer:

Here,


cos y = x cos (a + y), where cos a


Differentiating both sides with respect to x, we get





Multiplying the numerator and the denominator by cos(a+y) on th RHS we have,



[Given cos y = x cos (a + y)]


[ sin(a-b)=sina cosb - cosa sinb]



Hence Proved.



Question 66.

If prove that:


Answer:

Given:



Taking log on both sides we get,


log (x – y) +


(Using log ab = b log a and log (e) = 1)


Differentiating both sides we get,



Taking L.C.M and solving the equation we get,







Question 67.

If prove that


Answer:


Taking logon both sides,


Log x = log


log x = ……(i) [ Since log = a]


or, y = ……(ii)


Differentiating the given equation with respect to x,





[ From (i)]



[From (ii)]


Therefore,



Question 68.

If find


Answer:

Given








Question 69.

If find


Answer:

Given,



Using the theorem,



Here we have instead of x.


Hence, using the above theorem, we get,





Exercise 11.6
Question 1.

If prove that


Answer:

Here,


y


y


On squaring both sides,


x + y


Differentiating both sides with respect to x,





Hence proved.



Question 2.

If prove that


Answer:

Here,


y


y


On squaring both sides,


cos x + y


Differentiating both sides with respect to x,






Hence proved.



Question 3.

If prove that


Answer:

y


y


On squaring both sides,


log x + y


Differentiating both sides with respect to x,





Hence proved.



Question 4.

If prove that


Answer:

y


y


On squaring both sides,


tan x + y


Differentiating both sides with respect to x,





Hence proved.



Question 5.

If prove that


Answer:

Here,


y


y


By taking log on both sides ,


log y = log


log y = y(log sin x)


Differentiating both sides with respect to x by using product rule,







Hence proved.



Question 6.

If prove that at


Answer:

Here,


y


y


By taking log on both sides,


log y = log


log y = y(log tan x)


Differentiating both sides with respect to x using the product rule and chain rule,









Since 1}




Hence proved.



Question 7.

If prove that



Answer:

Here,


y


y = U + V + W


……(1)


Where, u ,v ,w


u


Taking log on both sides,


log u = log


log u


log u


Again, Taking log on both sides,


log log u = log


loglog u


Differentiating both sides with respect to x by using the product rule,





Put value of u and log u,


……(A)


Now,


v


taking log on both sides,


log v = log


log v


Differentiating both sides with respect to x by using the product rule,





Put value of v,


……(B)


Now ,


w =


taking log on both sides,


log w = log


log w


log w


taking log both sides,


log log w


Differentiating both sides with respect to x by using the product rule,





Put the value of w and log w,



Using equation A, B and C in equation (1),



Hence, proved.



Question 8.

If prove that


Answer:

Here,


y


y


By taking log on both sides,


log y = log


log y = y(log cos x)


Differentiating both sides with respect to x by using the product rule,











Exercise 11.7
Question 1.

Find , when

x = at2 and y = 2at


Answer:

Given that x = at2 , y = 2at


So,



Therefore,



Question 2.

Find , when

x = a(θ + sinθ) and y = a(1 – cosθ)


Answer:

x = a(θ + sinθ)


Differentiating it with respect to θ,


……(1)


And ,


y = a(1- cosθ)


Differentiating it with respect to θ ,



……(2)


Using equation (1) and (2) ,





{since, }




Question 3.

Find , when

x = acosθ and y = bsinθ


Answer:

as x = acosθ and y = bsinθ


Then,






Question 4.

Find , when

x = aeθ (sin θ – cos θ),y = aeθ(sinθ + cosθ)


Answer:

as x = aeθ (sin θ – cos θ)


Differentiating it with respect to θ



= a[eθ(cos θ + sin θ) + (sin θ-cos θ)eθ ]


……(1)


And , y = aeθ(sinθ + cosθ)


Differentiating it with respect to θ,



= a[eθ(cos θ - sin θ) + (sin θ + cos θ) eθ]


……(2)


Dividing equation (2) by equation (1),





Question 5.

Find , when

x = b sin2 θ and y = a cos2 θ


Answer:

as x = b sin2 θ


Then



And y = a cos2 θ





Question 6.

Find , when

x = a(1 – cos θ) and y = a(θ + sin θ) at


Answer:

as x = a(1 – cos θ)



And y = a(θ + sin θ)






Question 7.

Find , when

and


Answer:

as


Differentiating it with respect to t




……(1)


And


Differentiating it with respect to t,





……(2)


Dividing equation (2) by (1),





Question 8.

Find , when

and


Answer:

as


Differentiating it with respect to t using quotient rule,






……(1)


And


Differentiating it with respect to t using quotion rule





----(2)


Dividing equation (2) by (1),





Question 9.

Find , when

x = a(cosθ + θ sinθ) and y = a(sinθ – cosθ)


Answer:

the given equation are x = a(cosθ + θ sinθ)


Then




And y = a(sinθ – cosθ) so,





= a




Question 10.

Find , when

and


Answer:

as


Differentiating it with respect to using the product rule,






……(1)


And,


Differentiating it with respect to using the product rule,







……(2)


divide equation (2)by (1)





Question 11.

Find , when

and


Answer:

as,


Differentiating it with respect to t using quotient rule,






……(1)


And,


Differentiating it with respect to t using quotient rule,





----(2)


dividing equation (2)by (1),






Question 12.

Find , when

and


Answer:

as


Differentiating it with respect to t using chain rule ,






……(1)


Now ,


Differentiating it with respect to t using chain rule ,






……(2)


dividing equation (2) by (1),





Question 13.

Find , when

and


Answer:

as


Differentiating it with respect to t using quotient rule,





……(1)


And,


Differentiating it with respect to t using quotient rule,





……(2)


divided equation (2)by (1) so,





Question 14.

Find , when

If x = 2cos θ – cos 2θ and y = 2sin θ – sin 2θ, prove that


Answer:

as x = 2cos θ – cos 2θ


Differentiating it with respect to using chain rule ,




……(1)


And, y = 2sin θ – sin 2θ


Differentiating it with respect to using chain rule ,





……(2)


dividing equation (2)by equation (1),











Question 15.

Find , when

If x = ecos2 t and y = esin2t, prove that


Answer:

Here ,


Differentiating it with respect to using chain rule ,






……(1)


And,


Differentiating it with respect to using chain rule ,






……(2)


dividing equation (2)by (1),




[]



Question 16.

Find , when

If x = cos t and y = sin t, prove that at


Answer:

as x = cost


Differentiating it with respect to t ,



……(1)


And, y = sint


Differentiating it with respect to t,



……(2)


Dividing equation (2) by (1),










Question 17.

Find , when

If and prove that


Answer:

as


Differentiating it with respect to t,




……(1)


And


Differentiating it with respect to t,




……(2)


Dividing equation (2) by (1),






Question 18.

Find , when

If and -1 < t < 1, prove that


Answer:

as


Put t = tan




= 2


x = 2() [since, t = sin]


differentiating it with respect to t,


……(1)


Now ,



Put t = tan




= 2


[since t = tan]


differentiating it with respect to t,


……(2)


Dividing equation (2) by (1),





Question 19.

Find , when

If find


Answer:

as


Then




















Question 20.

Find , when

If find


Answer:

as


Differentiating it with respect to t using chain rule,




……(1)


And ,


Differentiating it with respect to t using chain rule,




……(2)


Dividing equation (2) by (1),





Question 21.

Find , when

If and find


Answer:

Here,


x = a()


differentiating bove function with respect to t, we have,






And



differentiating bove function with respect to t, we have,






| from equation 1 and 2




Question 22.

Find , when

If x = 10 (t – sin t), y = 12 (1 – cos t), find .


Answer:

Here, x = 10(t - sin t) y = 12(1-cos t)


……(1)


……(2)


| from equation 1 and 2





Question 23.

Find , when

If x = a(θ – sin θ) and y = a (1 + cos θ), find at


Answer:

Here,


x = () and y = a(1 + cos )


then ,





At x





Question 24.

Find , when

If x = a sin 2t (1 + cos 2t) and y = b cos 2t (1 – cos 2t), show that at


Answer:

considering the given functions,


x = asin 2t(1 + cos 2t) and y = b cos 2t(1-cos2t)


rewriting the above equations,


x = a sin 2tsin 4t


differentiating bove function with respect to t, we have,


……(1)


y = b cos 2t - b


differentiating above function with respect to t, we have,


……(2)


| from equation 1 and 2


At t




Question 25.

Find , when

If x = cos t (3 – 2 cos2 t) and y = sin t (3 – 2 sin2 t) find the value of at


Answer:

considering the given functions,


x = cost(3-2)


x = 3cos t -


……(1)


……(2)


| from equation 1 and 2




When t


cot



Question 26.

Find , when

If find


Answer:

: x, y






Question 27.

Find , when

If x = 3 sin t – sin 3t, y = 3 cost – cos 3t, find at


Answer:

x = 3sin t – sin 3t ,y = 3 cos t – cos 3t





When t





Question 28.

Find , when

If find


Answer:

sin x ,tan y


x









Exercise 11.8
Question 1.

Differentiate x2 with respect to x3.


Answer:

Let u = x2 and v = x3.


We need to differentiate u with respect to v that is find.


On differentiating u with respect to x, we get



We know




Now, on differentiating v with respect to x, we get



(using the same formula)



We have




Thus,



Question 2.

Differentiate log(1 + x2) with respect to tan–1x.


Answer:

Let u = log(1 + x2) and v = tan–1x.


We need to differentiate u with respect to v that is find.


On differentiating u with respect to x, we get



We know


[using chain rule]



However, and derivative of a constant is 0.





Now, on differentiating v with respect to x, we get



We know



We have





Thus,



Question 3.

Differentiate (log x)x with respect to log x.


Answer:

Let u = (log x)x and v = log x.


We need to differentiate u with respect to v that is find.


We have u = (log x)x


Taking log on both sides, we get


log u = log(log x)x


⇒ log u = x × log(log x) [∵ log am = m × log a]


On differentiating both the sides with respect to x, we get



Recall that (uv)’ = vu’ + uv’ (product rule)



We know and




But, u = (log x)x and





Now, on differentiating v with respect to x, we get




We have







Thus,



Question 4.

Differentiate with respect to cos–1 x, if

x ϵ (0, 1)


Answer:

Let and v = cos–1x.


We need to differentiate u with respect to v that is find.


We have


By substituting x = cos θ, we have




[∵ sin2θ + cos2θ = 1]


⇒ u = sin–1(sin θ)


(i) Given x ϵ (0, 1)


However, x = cos θ.


⇒ cos θ ϵ (0, 1)



Hence, u = sin–1(sin θ) = θ.


⇒ u = cos–1x


On differentiating u with respect to x, we get



We know



Now, on differentiating v with respect to x, we get




We have,





Thus,



Question 5.

Differentiate with respect to cos–1 x, if

x ϵ (–1, 0)


Answer:

Given x ϵ (–1, 0)

However, x = cos θ.


⇒ cos θ ϵ (–1, 0)



Hence, u = sin–1(sin θ) = π – θ.


⇒ u = π – cos–1x


On differentiating u with respect to x, we get




We know and derivative of a constant is 0.




Now, on differentiating v with respect to x, we get




We have





Thus,



Question 6.

Differentiate with respect to if



Answer:

Let and.


We need to differentiate u with respect to v that is find.


We have



By substituting 2x = cos θ, we have




[∵ sin2θ + cos2θ = 1]


⇒ u = sin–1(2 cos θ sin θ)


⇒ u = sin–1(sin2θ)


Given


However, 2x = cos θ ⇒






Hence, u = sin–1(sin 2θ) = π – 2θ.


⇒ u = π – 2cos–1(2x)


On differentiating u with respect to x, we get





We know and derivative of a constant is 0.






However,




Now, we have


On differentiating v with respect to x, we get




We know





We know and derivative of a constant is 0.





We have





Thus,



Question 7.

Differentiate with respect to if



Answer:

Let and.


We need to differentiate u with respect to v that is find.


We have



By substituting 2x = cos θ, we have




[∵ sin2θ + cos2θ = 1]


⇒ u = sin–1(2 cos θ sin θ)


⇒ u = sin–1(sin2θ)


Given


However, 2x = cos θ ⇒






Hence, u = sin–1(sin 2θ) = 2θ.


⇒ u = 2cos–1(2x)


On differentiating u with respect to x, we get




We know and derivative of a constant is 0.






However,




In part (i), we found


We have





Thus,



Question 8.

Differentiate with respect to if



Answer:

Let and.


We need to differentiate u with respect to v that is find.


We have



By substituting 2x = cos θ, we have




[∵ sin2θ + cos2θ = 1]


⇒ u = sin–1(2 cos θ sin θ)


⇒ u = sin–1(sin2θ)


Given


However, 2x = cos θ ⇒






Hence, u = sin–1(sin 2θ) = 2π – 2θ.


⇒ u = 2π – 2cos–1(2x)


On differentiating u with respect to x, we get





We know and derivative of a constant is 0.






However,




In part (i), we found


We have





Thus,



Question 9.

Differentiate with respect to if –1<x<1, x ≠ 0.


Answer:

Let and.


We need to differentiate u with respect to v that is find.


We have


By substituting x = tan θ, we have




[∵ sec2θ – tan2θ = 1]






But, cos2θ = 1 – 2sin2θ and sin2θ = 2sinθcosθ.





Given –1 < x < 1 ⇒ x ϵ (–1, 1)


However, x = tan θ


⇒ tan θ ϵ (–1, 1)




Hence,



On differentiating u with respect to x, we get




We know




Now, we have


By substituting x = tan θ, we have




[∵ sec2θ – tan2θ = 1]




⇒ v = sin–1(2sinθcosθ)


But, sin2θ = 2sinθcosθ


⇒ v = sin–1(sin2θ)


However,


Hence, v = sin–1(sin2θ) = 2θ


⇒ v = 2tan–1x


On differentiating v with respect to x, we get




We know




We have





Thus,



Question 10.

Differentiate with respect to if



Answer:

Let and.


We need to differentiate u with respect to v that is find.


We have


By substituting x = sin θ, we have




[∵ sin2θ + cos2θ = 1]


⇒ u = sin–1(2sinθcosθ)


⇒ u = sin–1(sin2θ)


Now, we have


By substituting x = sin θ, we have




[∵ sin2θ + cos2θ = 1]




Given


However, x = sin θ





Hence, u = sin–1(sin 2θ) = 2θ.


⇒ u = 2sin–1(x)


On differentiating u with respect to x, we get




We know




We have


Hence, v = sec–1(secθ) = θ


⇒ v = sin–1x


On differentiating v with respect to x, we get



We know



We have





Thus,



Question 11.

Differentiate with respect to if


Answer:

Let and.


We need to differentiate u with respect to v that is find.


We have


By substituting x = sin θ, we have




[∵ sin2θ + cos2θ = 1]


⇒ u = sin–1(2sinθcosθ)


⇒ u = sin–1(sin2θ)


Now, we have


By substituting x = sin θ, we have




[∵ sin2θ + cos2θ = 1]




Given


However, x = sin θ





Hence, u = sin–1(sin 2θ) = π – 2θ.


⇒ u = π – 2sin–1(x)


On differentiating u with respect to x, we get





We know and derivative of a constant is 0.




We have


Hence, v = sec–1(secθ) = θ


⇒ v = sin–1x


On differentiating v with respect to x, we get



We know



We have





Thus,



Question 12.

Differentiate (cos x)sin x with respect to (sin x)cos x.


Answer:

Let u = (cos x)sin x and v = (sin x)cos x.


We need to differentiate u with respect to v that is find.


We have u = (cos x)sin x


Taking log on both sides, we get


log u = log(cos x)sin x


⇒ log u = (sin x) × log(cos x) [∵ log am = m × log a]


On differentiating both the sides with respect to x, we get



Recall that (uv)’ = vu’ + uv’ (product rule)



We know and





We know




But, u = (cos x)sin x




Now, we have v = (sin x)cos x


Taking log on both sides, we get


log v = log(sin x)cos x


⇒ log v = (cos x) × log(sin x) [∵ log am = m × log a]


On differentiating both the sides with respect to x, we get



Recall that (uv)’ = vu’ + uv’ (product rule)



We know and





We know




But, v = (sin x)cos x




We have




Thus,



Question 13.

Differentiate with respect to if 0 < x < 1.


Answer:

Let and.


We need to differentiate u with respect to v that is find.


We have


By substituting x = tan θ, we have




[∵ sec2θ – tan2θ = 1]




⇒ u = sin–1(2sinθcosθ)


But, sin2θ = 2sinθcosθ


⇒ u = sin–1(sin2θ)


Given 0 < x < 1 ⇒ x ϵ (0, 1)


However, x = tan θ


⇒ tan θ ϵ (0, 1)




Hence, u = sin–1(sin2θ) = 2θ


⇒ u = 2tan–1x


On differentiating u with respect to x, we get




We know




Now, we have


By substituting x = tan θ, we have




[∵ sec2θ – tan2θ = 1]




⇒ v = cos–1(cos2θ – sin2θ)


But, cos2θ = cos2θ – sin2θ


⇒ v = cos–1(cos2θ)


However,


Hence, v = cos–1(cos2θ) = 2θ


⇒ v = 2tan–1x


On differentiating v with respect to x, we get




We know




We have





Thus,



Question 14.

Differentiate with respect to


Answer:

Let and.


We need to differentiate u with respect to v that is find.


We have


By substituting ax = tan θ, we have







On differentiating u with respect to x, we get




We know and derivative of a constant is 0.





We know




Now, we have


On differentiating v with respect to x, we get




We know





We know and derivative of a constant is 0.





We have





Thus,



Question 15.

Differentiate with respect to if


Answer:

Let and.


We need to differentiate u with respect to v that is find.


We have


By substituting x = sin θ, we have




[∵ sin2θ + cos2θ = 1]


⇒ u = sin–1(2sinθcosθ)


⇒ u = sin–1(sin2θ)


Given


However, x = sin θ





Hence, u = sin–1(sin 2θ) = 2θ.


⇒ u = 2sin–1(x)


On differentiating u with respect to x, we get




We know




Now, we have


By substituting x = sin θ, we have




[∵ sin2θ + cos2θ = 1]



⇒ v = tan–1(tanθ)


We have


Hence, v = tan–1(tanθ) = θ


⇒ v = sin–1x


On differentiating v with respect to x, we get



We know



We have





Thus,



Question 16.

Differentiate with respect to if 0 < x < 1.


Answer:

Let and.


We need to differentiate u with respect to v that is find.


We have


By substituting x = tan θ, we have




But,


⇒ u = tan–1(tan2θ)


Given 0 < x < 1 ⇒ x ϵ (0, 1)


However, x = tan θ


⇒ tan θ ϵ (0, 1)




Hence, u = tan–1(tan2θ) = 2θ


⇒ u = 2tan–1x


On differentiating u with respect to x, we get




We know




Now, we have


By substituting x = tan θ, we have




[∵ sec2θ – tan2θ = 1]




⇒ v = cos–1(cos2θ – sin2θ)


But, cos2θ = cos2θ – sin2θ


⇒ v = cos–1(cos2θ)


However,


Hence, v = cos–1(cos2θ) = 2θ


⇒ v = 2tan–1x


On differentiating v with respect to x, we get




We know




We have





Thus,



Question 17.

Differentiate with respect to if


Answer:

Let and v = sin–1(3x – 4x3)


We need to differentiate u with respect to v that is find.


We have


By substituting x = tan θ, we have





Given,


However, x = tan θ





As tan 0 = 0 and tan = 1, we have .


Thus, lies in the range of tan–1x.


Hence,



On differentiating u with respect to x, we get




We know and derivative of a constant is 0.




Now, we have v = sin–1(3x – 4x3)


By substituting x = sin θ, we have


v = sin–1(3sinθ – 4sin3θ)


But, sin3θ = 3sinθ – 4sin3θ


⇒ v = sin–1(sin3θ)


Given,


However, x = sin θ





Hence, v = sin–1(sin3θ) = 3θ


⇒ v = 3sin–1x


On differentiating v with respect to x, we get




We know




We have





Thus,



Question 18.

Differentiate with respect to sec–1 x.


Answer:

Let and v = sec–1x


We need to differentiate u with respect to v that is find.


We have



But, cos2θ = cos2θ – sin2θ and sin2θ = 2sinθcosθ.



[∵ sin2θ + cos2θ = 1]





Dividing the numerator and denominator with, we get








On differentiating u with respect to x, we get





We know and derivative of a constant is 0.




Now, we have v = sec–1x


On differentiating v with respect to x, we get



We know



We have





Thus,



Question 19.

Differentiate with respect to if –1 < x < 1.


Answer:

Let and.


We need to differentiate u with respect to v that is find.


We have


By substituting x = tan θ, we have




[∵ sec2θ – tan2θ = 1]




⇒ u = sin–1(2sinθcosθ)


But, sin2θ = 2sinθcosθ


⇒ u = sin–1(sin2θ)


Given –1 < x < 1 ⇒ x ϵ (–1, 1)


However, x = tan θ


⇒ tan θ ϵ (–1, 1)




Hence, u = sin–1(sin2θ) = 2θ


⇒ u = 2tan–1x


On differentiating u with respect to x, we get




We know




Now, we have


By substituting x = tan θ, we have




But,


⇒ v = tan–1(tan2θ)


However,


Hence, v = tan–1(tan2θ) = 2θ


⇒ v = 2tan–1x


On differentiating v with respect to x, we get




We know




We have





Thus,



Question 20.

Differentiate with respect to if .


Answer:

Let u = cos–1(4x3 – 3x) and


We need to differentiate u with respect to v that is find.


We have u = cos–1(4x3 – 3x)


By substituting x = cos θ, we have


u = cos–1(4cos3θ – 3cosθ)


But, cos3θ = 4cos3θ – 3cosθ


⇒ u = cos–1(cos3θ)


Given,


However, x = cos θ





Hence, u = cos–1(cos3θ) = 3θ


⇒ u = 3cos–1x


On differentiating u with respect to x, we get




We know




Now, we have


By substituting x = cos θ, we have




[∵ sin2θ + cos2θ = 1]



⇒ v = tan–1(tanθ)


However,


Hence, v = tan–1(tanθ) = θ


⇒ v = cos–1x


On differentiating v with respect to x, we get



We know



We have





Thus,



Question 21.

Differentiate with respect to if


Answer:

Let and.


We need to differentiate u with respect to v that is find.


We have


By substituting x = sin θ, we have




[∵ sin2θ + cos2θ = 1]



= tan–1(tanθ)


Given


However, x = sin θ




Hence, u = tan–1(tanθ) = θ


⇒ u = sin–1x


On differentiating u with respect to x, we get



We know



Now, we have


By substituting x = sin θ, we have




[∵ sin2θ + cos2θ = 1]


⇒ v = sin–1(2sinθcosθ)


⇒ v = sin–1(sin2θ)


However,


Hence, v = sin–1(sin 2θ) = 2θ.


⇒ v = 2sin–1(x)


On differentiating v with respect to x, we get




We know




We have





Thus,



Question 22.

Differentiate with respect to if 0 < x < 1.


Answer:

Let and


We need to differentiate u with respect to v that is find.


We have


By substituting x = cos θ, we have




[∵ sin2θ + cos2θ = 1]


⇒ u = sin–1(sinθ)


Given, 0 < x < 1 ⇒ x ϵ (0, 1)


However, x = cos θ


⇒ cos θ ϵ (0, 1)



Hence, u = sin–1(sinθ) = θ


⇒ u = cos–1x


On differentiating u with respect to x, we get



We know



Now, we have


By substituting x = cos θ, we have




[∵ sin2θ + cos2θ = 1]



⇒ v = cot–1(cotθ)


However,


Hence, v = cot–1(cotθ) = θ


⇒ v = cos–1x


On differentiating v with respect to x, we get



We know



We have





Thus,



Question 23.

Differentiate with respect to if


Answer:

Let and.


We need to differentiate u with respect to v that is find.


We have



By substituting ax = sin θ, we have




[∵ sin2θ + cos2θ = 1]


⇒ u = sin–1(2sinθcosθ)


⇒ u = sin–1(sin2θ)


Given


However, ax = sin θ





Hence, u = sin–1(sin 2θ) = 2θ.


⇒ u = 2sin–1(ax)


On differentiating u with respect to x, we get




We know





We know




Now, we have


On differentiating v with respect to x, we get




We know





We know and derivative of a constant is 0.





We have





Thus,



Question 24.

Differentiate with respect to if –1 < x < 1.


Answer:

Let and


We need to differentiate u with respect to v that is find.


We have


By substituting x = tan θ, we have





Given, –1 < x < 1 ⇒ x ϵ (–1, 1)


However, x = tan θ


⇒ tan θ ϵ (–1, 1)




Hence,



On differentiating u with respect to x, we get




We know and derivative of a constant is 0.




Now, we have


On differentiating v with respect to x, we get




We know





We know and derivative of a constant is 0.





We have





Thus,




Mcq
Question 1.

Choose the correct alternative in the following:

If f(x) = logx2 (log x), then f’(x) at x = e is

A. 0

B. 1

C. 1/e

D 1/2e


Answer:

f(x) = logx2 (log x)

Changing the base, we get







Putting x = e, we get




(∵ log e = 1)


(∵ log 1 = 0



Question 2.

Choose the correct alternative in the following:

The differential coefficient of f(log x) with respect to x, where f(x) = log x is

A.

B.

C. (x log x)–1

D. none of these


Answer:

Given: f(x) = log x

∴ f(log x) = log(log x)




∴ f(log x) = (x log x)-1


Question 3.

Choose the correct alternative in the following:

The derivative of the function at x = π/6 is

A. (2/3)1/2

B. (1/3)1/2

C. 31/2

D. 61/2


Answer:



Putting x = π/6, we get





Simplifying above we get




∴ f’(x) = √3 = (3)1/2


Question 4.

Choose the correct alternative in the following:

Differential coefficient of sec (tan–1 x) is

A.

B.

C.

D.


Answer:

Let f(x) = sec (tan–1 x)


Let θ = tan-1x


--(1)



-- From (1)


Now θ = tan-1x


= x = tan θ


∵ sec2θ – tan2θ = 1


Putting values, we get





Question 5.

Choose the correct alternative in the following:

If f(x) = tan–1 0 ≤ x ≤ π/2, then f’ (π/6) is

A. –1/4

B. –1/2

C. 1/4

D. 1/2


Answer:



∵ sin2x = 2 sin x cos x


⇒ sin x = 2 sin x/2 cos x/2



∵ sin2x/2 + cos2x/2 = 1




Dividing by cos x/2 we get


Taking – common





∵ 0 ≤ x ≤ π/2




Question 6.

Choose the correct alternative in the following:

If

A.

B.

C.

D.


Answer:


Taking log both sides we get




Differentiating w.r.t x we get,




Putting value of y, we get



Question 7.

Choose the correct alternative in the following:

If xy = ex–y, then is

A.

B.

C. not defined

D.


Answer:

xy = ex–y

Taking log both sides we get


log xy = log ex–y


y log x = (x-y) loge


y log x = (x-y) ∵ loge = 1



Differentiating w.r.t x we get,




Question 8.

Choose the correct alternative in the following:

Given f(x) = 4x8, then

A.

B.

C.

D.


Answer:

f(x) = 4x8

f'(x) = 32x7


Consider option (A)





Consider option (B)





Consider option (C)





Consider option (D)





Question 9.

Choose the correct alternative in the following:

If x = a cos3 θ, y = a sin3 θ, then

A. tan2 θ

B. sec2 θ

C. sec θ

D. |sec θ|


Answer:

We are given that




Now, we know



Now,



(Using Chain Rule)


Again



(Using Chain Rule)


Now,


By Simplifying we get,





Question 10.

Choose the correct alternative in the following:

If

A.

B.

C.

D.


Answer:


Put x = tan θ ⇒ θ = tan-1x






Putting value of θ we get,



Differentiating w.r.t x we get,





Question 11.

Choose the correct alternative in the following:

The derivative of with respect to at x = –1/3

A. does not exist

B. 0

C. 1/2

D. 1/3


Answer:



Considering u,



Put x = cos θ


θ = cos-1x ----(1)




From (1)


Differentiating w.r.t x



Considering v,



Differentiating w.r.t x








Question 12.

Choose the correct alternative in the following:

For the curve .. at (1/4, 1/4) is

A. 1/2

B. 1

C. –1

D. 2


Answer:


Differentiating w.r.t x we get,







Question 13.

Choose the correct alternative in the following:

If sin (x + y) = log (x + y), then

A. 2

B. –2

C. 1

D. –1


Answer:

sin (x + y) = log (x + y)

Differentiating w.r.t x we get,








Question 14.

Choose the correct alternative in the following:

Let and

A. 1/2

B. x

C.

D. 1


Answer:

We are given that




Now, we know



Now,



Put x = tan θ


θ = tan-1x ----(1)






Again



Put x = tan θ


θ = tan-1x ----(1)





-- From (1)


Now,



Question 15.

Choose the correct alternative in the following:

equals

A. 1/2

B. –1/2

C. 1

D. –1


Answer:



--(1)



∵ sin2t = 2sint.cost and 1+cos2t = 2cos2t



--From (1)



Question 16.

Choose the correct alternative in the following:

equals

A.

B. 1

C.

D.


Answer:


-----(1)





∵ log e = 1



--From (1)





Question 17.

Choose the correct alternative in the following:

If

A.

B.

C.

D.


Answer:


Squaring both sides



Differentiating w.r.t x we get,





Question 18.

Choose the correct alternative in the following:

If 3 sin(xy) + 4cos (xy) = 5, then

A.

B.

C.

D. none of these


Answer:

3 sin(xy) + 4cos (xy) = 5

Differentiating w.r.t x we get,



(Using Chain Rule)






Question 19.

Choose the correct alternative in the following:

If sin y = x sin (a + y), then is

A.

B.

C. sin a sin2 (a + y)

D.


Answer:

sin y = x sin (a + y)


Differentiating w.r.t y we get,









Question 20.

Choose the correct alternative in the following:

The derivative of cos–1 (2x2 – 1) with respect to cos–1 x is

A. 2

B.

C. 2/x

D. 1 – x2


Answer:

Let u = cos–1 (2x2 – 1) and v = cos–1 x


Considering u = cos–1 (2x2 – 1)


Put x = cos θ ⇒ θ = cos-1x ---(1)


u = cos–1 (2cos2θ – 1)


u = cos–1 (cos2θ) ∵ 2cos2θ – 1 = cos2θ


u = 2θ


u = 2 cos-1x -- From(1)


Differentiating w.r.t x we get,



Considering v = cos–1 x


Differentiating w.r.t x we get,






Question 21.

Choose the correct alternative in the following:

If then f’(x) is equal to

A. 1 for x < –3

B. –1 for x < –3

C. 1 for all xϵ R

D. none of these


Answer:






Question 22.

Choose the correct alternative in the following:

If f(x) = |x2 – 9x + 20|, then f’(x) is equal to

A. –2x + 9 for all xϵ R

B. 2x – 9 if 4 < x < 5

C. –2x + 9 if 4 < x < 5

D. none of these


Answer:

f(x) = |x2 – 9x + 20|

= |x2 – 4x – 5x + 20|


= |x(x – 4) – 5(x – 4)|


f(x) = | (x –5) (x – 4) |




Question 23.

Choose the correct alternative in the following:

If then the derivative of f(x) in the interval [0, 7] is

A. 1

B. –1

C. 0

D. none of these


Answer:







Since there is no fixed value of f’(x) in the interval [0,7], so the answer is (D) none of these


Question 24.

Choose the correct alternative in the following:

If f(x) = |x – 3| and g(x) = fof(x), then for x > 10, g’(x) is equal to

A. 1

B. –1

C. 0

D. none of these


Answer:


= ||x – 3| – 3|



Since we have given x > 10 then |x – 3| = (x – 3)


∴ g(x) = |(x – 3) – 3| = |x – 6|



Since we have given x > 10 then |x – 6| = (x – 6)


∴ g(x) = (x – 6)



Question 25.

Choose the correct alternative in the following:

If then f’(x) is equal to

A. 1

B. 0

C. xℓ+m+n

D. none of these


Answer:





Differentiating w.r.t x


� �


Question 26.

Choose the correct alternative in the following:

If, .. then is equal to

A. 1

B.

C. 0

D. none of these


Answer:





Differentiating w.r.t x



Question 27.

Choose the correct alternative in the following:

If then is equal to

A.

B.

C.

D. none of these


Answer:


Let x3 = cos p and y3 = cos q


cos-1x3 = p and cos-1y3 = q --- (1)





Comparing L.H.S and R.H.S we get,







Substituting value of p and q from (1)



Differentiating w.r.t x we get,




Comparing L.H.S and R.H.S we get



Question 28.

Choose the correct alternative in the following:

If then the value of at is given by

A. ∞

B. 1

C. 0

D. 1/2


Answer:




Differentiating w.r.t x we get,






Question 29.

Choose the correct alternative in the following:

If is equal to

A.

B.

C.

D. none of these


Answer:



Put y = x tanθ


----(1)








Taking tan on both sides




Differentiating w.r.t x we get,





Question 30.

Choose the correct alternative in the following:

If sin y = x cos(a + y), then is equal to

A.

B.

C.

D. none of these


Answer:

sin y = x cos(a + y)


Differentiating w.r.t y we get,


(Using quotient rule)




Using cos(a-b) = cos a.cos b + sin a.sin b




Question 31.

Choose the correct alternative in the following:

If

A.

B.

C.

D.


Answer:


(Using quotient rule)






Question 32.

Choose the correct alternative in the following:

If equals.

A.

B.

C.

D.


Answer:


Squaring both sides, we get


y2 = sinx + y


Differentiating w.r.t y we get





Question 33.

Choose the correct alternative in the following:

If then is equal to

A.

B. 0

C. 1

D. none of these


Answer:


Dividing Numerator and denominator by cos x we get,







Differentiating w.r.t x we get,